You are on page 1of 84

Investment

Planning
(Workbook)

IMS Learning Resources Pvt Ltd.


E-Block, 6th Floor,
NCL Bandra Premises,
Bandra Kurla Complex,
Bandra (E). Mumbai 400 051
Tel No: +91 22 66680005
Fax No: +91 22 66680006
Email: help.fp@imsindia.com
Website:www.imsindia.com

PREFACE

Having gained conceptual clarity through concept book and in-class experience, it is
important that you apply and strengthen your fundamentals; the exercises in the
workbook will offer you practice and challenge your ability and understanding.
The questions have been designed in such a way that each test covers all the concepts
discussed in the book.

The Tests will help you identify your conceptual gap; assess your strengths and
areas for improvement.

If you need any support in your preparation; Do write to us at help.fp@imsindia.com


In addition to these workbooks, you can also access more practice tests online. For
details write to us at help.fp@imsindia.com

Investment Planning

Workbook

1.

The bid-asked spread is best described by


which one of the following statements?

6.

a) It is the broker's commission.


b) It is the dealer's gross income from a
transaction.
c) It is larger for illiquid securities than it is
for liquid securities.
d) All of the above are true.
e) None of the above.
2.

3.

8.

a)
b)
c)
d)

Workbook

It has no voting privileges.


It receives no cash dividends.
It may be resold at any time.
All of the above.

Which one of the following equations


correctly defines the dividend yield (y) from
a share of common stock?
a) y = (purchase price) + (cash dividend, if
any) / purchase price
b) y = (price change) + (cash dividend, if
any) / purchase price
c) y = price change /purchase price
d) y = cash dividend (if any) / purchase
price

One Rupee, Re.1


One percentage point, 1 percent
One paisa, Re.0.01
One one-hundredth of one percentage
point, 0.01 of 1 percent,

Which one of the following statements best


describes a Government bond?

Preference shareholders receive priority


over common stockholders with respect to
which of the following?
a) Dividends cannot be paid to common
stockholders unless the preference
stockholders receive their stated
dividend.
b) In the event of bankruptcy and liquidation,
the preference shareholders are paid
before the common shareholders.
c) Preference shareholders get to elect the
Chairman of the corporation's Board of
Directors.
d) Both (a) and (b) are true, but (c) is false.
e) All of the above are true.

Which one of the following is a basis point?


a)
b)
c)
d)

5.

7.

Which of the following are characteristics


of money market securities?
a) They are issued by the Government,
municipalities and large corporations
that have high-quality ratings.
b) All have terms to maturity that are 270
days or less.
c) All tend to have large amounts of
purchasing power risk.
d) Both (a) and (b)
e) Both (b) and (c)

4.

a) They can legally demand information from


a corporation in which they are a
shareholder and thus gain access to its
books.
b) They can vot e for the common
shareholders' dividend.
c) They can vote for the preference
shareholders' dividend.
d) All of the above.

How do T -bills pay interest to their


investors?
a) Coupon interest.
b) Possible price appreciation above their
discounted price.
c) T-bills pay no interest.
d) Difference between discount price and
face value.

Which of the following rights do equity


shareholders have?

9.

A preference share ___________


a)
b)
c)
d)
e)

Investment Planning

pays fixed dividend


is a marketable security
is a debt security
both (a) and (b)
all of the above

10.

a) Some issues of preference share may


be converted into common share at the
option of the investor any time and at a
conversion ratio that never changes.
b) Some issues of preference share may
be converted into common share at the
option of the investor within a limited
number of years after the preferred stock
is issued.
c) Some issues of preference share may
be converted into common share at the
option of the investor only after a
specified number of years have elapsed
since the preference share was initially
issued,
d) All of the above are true.
e) Preference share is never a convertible
security.
11.

The quality ratings of a corporation's bond


issue are primarily determined by which of
the following?
a) The level and trend of the issuer's
financial ratios
b) The level and structure of interest rates
c) The issuer's financial condition and the
indenture contract that governs the
issuing firm
d) All the above

13.

14.

One bond with an AA-grade rating might pay


a higher yield-to-maturity than another AAgrade bond issued at a different time by the
same corporation because of which of the
following reasons?
a) Bonds with longer maturities normally
pay higher rates of interest than similar
bonds that have shorter maturities.
b) The bond market is sometimes irrational
and evaluates the riskiness of some
bond issues erroneously.

Which of the following bond quality ratings


applies to default-free bonds?
a)
b)
c)
d)

15.

AAA.
AA.
Both b and a are default-free.
None of the above is default-free.

A security will not earn the yield-to-maturity


that was promised when the security was
purchased if the following conditions
occurs?
a) The issuer defaults on either the interest
or principal payments.
b) The investor sells the security prior to
its maturity date.
c) Cash flows from the security paid to the
investor prior to its maturity date are held
in cash or spent on consumption goods
rather than reinvested.
d) All of the above are true.
e) None of the above are true.

Which one of the following statements best


describes corporate bonds?
a) Bond investors are creditors of the
corporation.
b) All the bonds make coupon interest
payments once per annum.
c) Both (a) and (b) are true.
d) None of the above are true.

12.

c) One bond issue is a secured one


whereas the other issue is unsecured.
d) All of the above

Which of the following statements best


describes the convertibility of preference
share?

16.

An investor that employed a naive buy-andhold strategy would be employing a passive


investment management strategy.
a) True
b) False

17.

An index fund is best described by which of


the following?
a) A mutual fund constructed to achieve a
particular investment goal.
b) A portfolio that attempts to match the
performance of some stock market index
by investing in the same stocks and in
the same proportions as those that
comprise the selected market index.
c) Both of the above.
d) An investment portfolio that appreciates
in value at least as rapidly as some
inflation index (such as the Consumer
Price Index, for instance).
e) Both (a) and (d).

Investment Planning

Workbook

18.

The weights used in constructing a valueweighted stock market index are best
described by which of the f ollowing
statements?
a) Equal weights are assigned to every
security in the index.
b) The weight assigned to each stock is
proportionate to its price per share.
c) The share price of every stock in the
index is multiplied by the number of
shares outstanding to determine the
weight of that issue based on its total
value stated as a proportion of the
aggregate market value of all the stocks
in the index.
d) The weight assigned to each stock in the
index is proportional to the number of
shares that issue has outstanding stated
as a proportion of the aggregate number
of shares outstanding for all issues that
comprise the index.
e) Both (a) and (d) are true.

19.

21.

23.

24.

a) True
b) False

Workbook

If a corporation has preference share


outstanding its book value per share equals
its total net worth divided by the number of
shares of common stock plus preference
shares it has outstanding.
a) True
b) False

25.

Which of the following ratios will increase


as a firm uses more financial leverage?
a)
b)
c)
d)
e)

26.

The times-interest-earned ratio


The debt-to-equity ratio
The inventory turnover
Both (a) and (b)
Both (a) and (c)

Which of the following factors tends to


increase the growth rate of a corporation?
a)
b)
c)
d)
e)

27.

When a firm pays creditors the transaction


does not affect the equity capital shown in
its balance sheet in any way.

The retention rate equals 100 percent less


the percent of the corporation's earnings
paid out for cash dividends.
a) True
b) False

Depreciation must be entirely omitted from


a firm's net profit in order to determine how
much cash flow the firm generated.
a) True
b) False

A primary issue of bonds or stock would


increase bot h sides of the issuing
company's balance sheet by the same
amount.
a) True
b) False

All stock market indexes are most


accurately characterized by which of the
following statements about the degree to
which they covary together?
a) They are perfectly positively correlated.
b) They are highly positively correlated.
c) They are uncorrelated.
d) They are negatively correlated.
e) It is impossible to generalize, some are
highly positively correlated and some are
negatively correlated.

20.

22.

External borrowing
Increasing the retention rate
Increasing the rate of return on equity
Both (a) and (b)
All of the above

A company has total assets of 2,000,000. It


has 700,000 in long-t erm debt. If
stockholders' equity is 900,000, what is its
total debt to total asset ratio?
a)
b)
c)
d)
e)

Investment Planning

45
47
59
52
55

percent
percent
percent
percent
percent

28.

A corporation had a total debt to total asset


ratio of .4, total debt of Rs.200,000, and net
income of Rs.30,000. Determine the
corporation's return on equity.
a)
b)
c)
d)
e)

29.

30.

Imperfect hedges occur when either the


quantities sold short and bought long are
out of balance, or the purchase and short
sale prices differ.
a) True
b) False

Speculating
Hedging
Arbitrage
All the above

Investors' motives are not correctly


described by which of the f ollowing
statements?
a) Having long positions indicates that the
buyer is bullish.
b) Uncovered short sellers expect the
market to be bearish.
c) Hedgers are always bullish.
d) Both (a) and (b) are false
e) All of the above are false

An increase in inventory
An increase in accounts receivable
An increase in investments
An increase in accounts payable
None of the above

Assume the correlation coefficient r between


the rates of return from these two cement
sector shares, say, A and G was +0.9. If
you took a long position in G and a short
position in A (or vice versa) of exactly equal
value you would be perfectly hedged.

Selling securities short is useful in which of


the following activities?
a)
b)
c)
d)

35.

a) True
b) False
32.

34.

Which of the following is a source of funds?


a)
b)
c)
d)
e)

31.

Rs.75
Rs.100
Rs.110
Rs.115
Rs.117

Which of the following statements correctly


describes a speculator's (short seller)
profits?
a) The per share profit is limited to an
amount equal to the price at which the
shares were sold short.
b) The short seller earns Re.1 profit for
every Re.1 fall in price of the security.
c) Short selling can help arbitragers earn
profits.
d) All the above are true.

8 percent
9 percent
10 percent
12 percent
14 percent

Assume the f ollowing information:


Stockholders' Equity = Rs.2,000; Shares
Outstanding = 40; Market Price to book
value = 2. Determine the market price for
the firm's common stock.
a)
b)
c)
d)
e)

33.

36.

Why do share prices usually drop when


news about decline in a company's earnings
per share is reported?
a) Because a reduction in a earnings
means that the firm has less money with
which to pay dividends and therefore the
market f ears a reduction in the
company's future dividends.
b) Because the share market anticipates
that a decreased level of earning power
might be the indicator of default and
perhaps even bankruptcy.
c) The statement is false. Share prices do
not usually react to announcements
about current earnings.
d) Both (a) and (b) are true.

Investment Planning

Workbook

37.

Interest-rate risk is defined by which of the


following statements?

40.

a) Fluctuations in the coupon interest rates


from one bond issue to the next
b) Fluctuations in the market prices of
bonds as their prices move inversely to
the prevailing market interest rates
c) The variability of returns as a result of
fluctuations in market interest rates
d) Both (a) and (b)
e) All of the above
38.

Assume that you are an investment adviser


and one of your clients, on your advice,
invested Rs.100,000 in Treasury bonds due
to mature in 2 years. If your client becomes
worried that a general increase in the level
of interest rates will reduce the market value
of his bond portfolio, what should you say
to allay your client's fears?
a) You could assuage your client's fears by
claiming you foresee only stable interest
rates ahead.
b) You could instruct your client to liquidate
their portfolio of Treasury bonds and
reinvest the proceeds in a bank.
c) Both (a) and (b) are true.
d) You could tell your client not to worry
because the market prices of short-term
bonds do not fluctuate very much.

39.

Assume you are an financial advisor and


one of your clients reads something about
interest-rate risk and is worried that if market
interest rates declined his coupon interest
income will likewise decline. His bond
investments have maturities ranging from
15 to 30 years. What advice is appropriate
for this client?
a) Tell the investor to liquidate her couponpaying bonds and reinvest the money in
zero coupon bonds.
b) Tell your client not to worry, her coupon
income will not vary until her coupon
bonds mature in 15 to 30 years.
c) Both (a) and (b) are true.
d) The client need not worry if market
interest rates are expected to rise
because coupon rates vary inversely
with market interest rates and therefore
her coupon interest could increase.
e) All the above are true.

Workbook

Calculate the (1) expected rate of return,


E(r), from the probability distribution of
returns below for the ABC common share.

The expected rate of return for ABC is which


one of the following?
a)
b)
c)
d)
e)
41.

The E(r) is 5 percent.


The E(r) is 20 percent.
The E(r) is 5 percent.
The E(r) is 10 percent.
The E(r) is 12 percent.

Which one of the following statements


describes the phrase 'risk'?
a) The phrase total risk is synonymous with
the variability of return from an asset.
b) Bond quality ratings essentially measure
the probability that an issue of bonds falls
into default.
c) Although Treasury bonds are free from
default risk they nevertheless contain
substantial amounts of interest-rate risk.
d) Both (a) and (b) are true.
e) All the above are true.

42.

If ABS's price is Rs.40 per share and its


current dividend of Rs.3.85 per share, which
is growing at a 7 percent rate per year,
determine its required return?
a)
b)
c)
d)
e)

Investment Planning

16.2 percent
15.1 percent
16.6 percent
17.3 percent
18.2 percent

43.

If ABS's pays dividend of Rs.3.85 per share


which is growing at a 7 percent rate per year
and is expected to grow at the same rate in
future. Its required rate of return is 14.5%
Determine its share value.
a)
b)
c)
d)
e)

44.

48.

Rs.51.33
Rs.55.08
Rs.57.02
Rs.52.05
Rs.50.75

49.

The value of an option tends to increase as


the volatility (or risks) of the underlying asset
increases.

If you purchase a put option, you are


expecting the value of the underlying asset
to increase.
a) True
b) False

51.

Each contract is equal to 100 shares. .

Rs.1,000
Rs.1,500
-Rs.2,000
-Rs.4,000
-Rs.500

If you had purchased five 3-month puts on


C, what would your profit "or loss position
have been at maturity if the share's price
was Rs.32?
a)
b)
c)
d)
e)

Directions for Questions 47 to 55: Use the


information given in the table below to answer the
questions. Ignore taxes and transaction costs.

Rs.2,000
Rs.5,700
Rs.8,200
Rs.4,000
Rs.3,600

If you had purchased five 3-month call


options of C and the price of C's share is
Rs.32 at maturity. Determine your profit or
loss on the investment.
a)
b)
c)
d)
e)

50.

Rs.200
Rs.400
-Rs.460
Rs.500
Rs.560

If B's price is Rs.35 at the maturity of the 6month option, determine the value of
purchase of five 6-month put contracts at
their maturity date.
a)
b)
c)
d)
e)

a) True
b) False
46.

If you purchase one 3-month call contract


on A, what profit or loss will you make on
the maturity date if the price of A at that
time is Rs.57?
a)
b)
c)
d)
e)

A company has current earnings per share


of Rs.6. Assume a dividend-payout ratio of
55 percent. Earnings grow at a rate of 8.5
percent per year. If the required rate of
return is 15 percent, what is its current
value?
a)
b)
c)
d)
e)

45.

Rs. 52.48
Rs. 49.25
Rs. 54.93
Rs. 55.75
Rs. 47.26

47.

-Rs.225
-Rs.400
-Rs.600
Rs.400
Rs.600

Your client wrote five 6-month call options


on B's share. What is his profit or loss on
the options at maturity if the price of B at
that time is Rs.43?
a)
b)
c)
d)
e)

Investment Planning

Rs.625
-Rs.600
Rs.400
Rs.300
Rs.200

Workbook

52.

If your client had written five 6-month put


options on B, what would his profit or loss
have been at the maturity of the options if
the share price was Rs.43 per share?
a)
b)
c)
d)
e)

53.

58.

If an investor is bearish on a share, buying a


put is usually better than selling short because
a) The holder's losses can be no more than
the put premium if the share price rises,
but the short seller's losses could be
unlimited in this situation.
b) The short sale will become worthless
after a short period of time but the put
will not become worthless.
c) The short seller must pay any dividends
paid by the security the short seller
borrowed.
d) (a) and (b).
e) None of the above

55.

Call option-premiums for a given asset tend


to increase when :
a) The price of the underlying asset
decreases.
b) The volatility of the underlying asset
decreases.
c) The time to maturity of the option
increases.
d) Both (a) and (b).
e) None of the above.

56.

Dividend payment on a firm's common


share tends to lower the value of a call
option on the firm's equity.
a) True
b) False

59.

A's 3-month call


B's 6-month put
C's 6-month put
a and b
None of the above

When a firm's dividend payment is included


in the Black Scholes model, the value of a
put decreases.
a) True
b) False

Which of the following options are in the


money?
a)
b)
c)
d)
e)

54.

Rs.1,000
Rs.2,000
Rs.1,800
Rs.1,500
-Rs.500

57.

Option prices that are calculated with the


Black Scholes model are not very sensitive
to changes in the asset's standard deviation
of returns.
a) True
b) False

60.

The option values calculated with the


binomial model will approach those
calculated with the Black Scholes model for
a given period of time as we divide the fixed
time period into smaller and smaller units.
a) True
b) False

Directions for Questions 61 to 70: Using the


information given below, answer the questions with
the Black Scholes option pricing model.
The options on the share of National Corporation
have the following values:
S = Rs.55 k = Rs.52 R = .10 u = .33 and T = .4
Assume that no dividends are currently being paid.
61.

What is the value of a call on the National


Corporation?
a)
b)
c)
d)
e)

Rs.6.50
Rs.6.80
Rs.7.33
Rs.8.05
Rs.8.35

The Black Scholes model cannot be used to


determine the overall market value of a firm.
a) True
b) False

Workbook

Investment Planning

62.

If the call is undervalued, what approach


should an investor follow?
a) Buy 1,000 calls, sell short 733 shares of
National share.
b) Buy 1,000 calls, sell short 600 shares of
National share.
c) Buy 500 shares of National, sell 1,000
calls.
d) Buy 714 shares of National, sell 1,000
calls.
e) Buy 600 shares of National, sell 1,000
calls.

63.

Rs.1.75
Rs.2.29
Rs.2.65
Rs.2.95
Rs.3.15

65.

71.

Determine the price of a put on National's


share with a 4 percent annual dividend.
a)
b)
c)
d)
e)

10

Rs.5.57
Rs.5.85
Rs.6.03
Rs.6.35
Rs.6.74

Rs.2.56
Rs.2.85
Rs.1.95
Rs.2.40
Rs.2.96

Rs.1.50
Rs.1.75
Rs.1.91
Rs.2.51
Rs.2.30

If k = Rs.54, determine the value of a call


on National's share if the other inputs do
not change. Assume no dividends.
a)
b)
c)
d)
e)

Assume the National Corporation is paying


dividends at the rate of 4 percent per year.
Determine the price of a call.

Rs.6.80
Rs.7.25
Rs.7.61
Rs.7.90
Rs.8.10

If T = .5, determine the value of a put on


National's share if the other inputs do not
change. Assume no dividends.
a)
b)
c)
d)
e)

70.

.37
.40
.25
.35
.18

If R rises to .12, determine the price of a


call on National's share if the other inputs
do not change. Assume no dividends.
a)
b)
c)
d)
e)

If the put on National's share is overpriced,


what should an investor do?

a)
b)
c)
d)
e)
66.

68.

69.

a) Sell 1,000 puts for every 400 shares sold.


b) Buy 1,000 puts for every 300 shares
purchased.
c) Sell 1,000 puts for every 229 shares sold.
d) Sell 1,000 puts for every 350 shares
purchased.
e) Buy 1,000 puts for every 400 shares
sold.

If the price of a National call is Rs.6.41,


determine the implied standard deviation of
returns without dividends.
a)
b)
c)
d)
e)

Determine the value of a put on National's


share.
a)
b)
c)
d)
e)

64.

67.

Rs.5.86
Rs.6.19
Rs.6.30
Rs.6.42
Rs.6.76

Wit h two assets, as the correlat ion


coefficient between the two assets is
reduced, the portfolio's risk is reduced.
a) True
b) False

72.

A 50-asset portfolio has _______ unique


covariance terms.
a)
b)
c)
d)

Investment Planning

2499
1449
1225
none of the above

Workbook

73.

In the two-asset case, the portfolio risk-return


possibilities are nonlinear when the
correlation between the asset returns is less
than + 1.

77.

a)
b)
c)
d)
e)

a) True
b) False
Directions for questions 74 and 75: Refer to the
data below.

Calculate the covariance of returns.


a)
b)
c)
d)
e)

75.

220.32
-420.11
145.22
270.36
162.08

78.

Calculate the correlation coefficient from the


above information.
a)
b)
c)
d)
e)

.540
.869
.923
.758
.697

79.

l
l
l

76.

Determine the risk of a portfolio of 25


percent M and 75 percent N.
a)
b)
c)
d)
e)

Workbook

18.25 percent
30.15 percent
24.15 percent
21.75 percent
27.13 percent

80.

0.63
.68
.91
1.05
1.10

Determine the Sharpe performance index


for the XYZ Fund for the 5-year period.
a)
b)
c)
d)
e)

Two companies, M and N have the following risk


and return statistics:
Standard deviation (M) = 18%;
Standard deviation (N) = 30%
Expected Return (M) = 14%; Expected
Return (N) = 19%
Correlation coefficient = 0.28

Determine the Sharpe performance index


for the ABC Fund for the 5-year period.
a)
b)
c)
d)
e)

Directions for questions 76 and 77: Refer to the


data below :

19.5 percent
21.7 percent
17.8 percent
23.0 percent
25.4 percent

Directions for questions 78 to 80: Use the


information in the table below to answer the
questions. Assume the risk less rate is the average
of the five annual T-bill returns.

The post tax returns of 2 shares are:

74.

Determine risk of a portfolio of 50% M and


50% N from the information given in
problem 81.

.48
0.34
.78
.92
1.05

Det ermine the Jensen perf ormance


measure (alpha) for the ABC Fund over the
5-year period.
a)
b)
c)
d)
e)

Investment Planning

4.27
2.69
1.76
2.01
3.76

11

81.

Duration for a zero coupon bond is less than


its term to maturity.

90.

a) True
b) False
82.

83.

a)
b)
c)
d)
e)

Longer-term bonds are almost always more


volatile in terms of price than short-term
bonds for a given change in interest rates.
a) True
b) False

91.

Bond price volatility is directly related to the


bond's coupon.

Duration for a coupon-paying bond is always


less than its term to maturity.
92.

a) True
b) False
85.

a) True
b) False
86.

93.

There is a direct relationship between a


bond's coupon and duration.

87.

As a bond's YTM increases, if other things


are held constant, its duration decreases.
a) True
b) False

88.

When a bond is selling at a discount, its YTM


exceeds the coupon rate.
a) True
b) False

89.

When a bond's YTM equals its coupon rate,


the bond's price is less than par value.

94.

The time structure of a bond's cash flows


The bond's interest-rate risk
Both a and b above
The default risk of the bond issue
None of the above

If the market rate of interest falls, a couponpaying bond will _________.


a)
b)
c)
d)
e)

a) True
b) False

Rs.1,130.55
Rs.935
Rs.757
Rs.868
Rs.898.94

A bond's duration measures which one of


the following?
a)
b)
c)
d)
e)

For any given maturity, bond price


movements that result from an equal
absolute decrease or increase in the yieldto-maturity are symmetrical.

Rs.1,051.65
Rs.1,159.88
Rs.885.30
Rs.888.89
Rs.955.41

The price of the bond in above question after


2 years, assuming everything else stays the
same, is (Hint:There will be 8 years until
maturity.)
a)
b)
c)
d)
e)

a) True
b) False
84.

10 percent semiannual bond with a YTM of


12 percent and 10 years to maturity has a
price equal to ____________.

Decrease in value
Experience a decrease in duration
Experience an increase in duration
None of the above
Both a and b above

A bond's reinvestment rate risk:


a) Refers to the problem of being able to
purchase another bond with the same
or higher YTM when the existing bond
matures or is called.
b) Is the risk of not being able to reinvest
the coupons of a bond at the bond's
YTM.
c) Is the same as marketability risk.
d) Both (a) and (b).
e) None of the above.

a) True
b) False

12

Investment Planning

Workbook

95.

If you expect a large decline in interest rates,


which of the following investments should
you choose?
a)
b)
c)
d)
e)

96.

Money market fund


Low-coupon short-term bond
High-coupon short-term bond
Long-term zero coupon bond
Short-term zero coupon bond

Bonds with higher coupons, other things


being the same,
a) have more interest-rate risk than bonds
with smaller coupons
b) have less interest-rate risk than bonds
with smaller coupons
c) have higher duration than smallercoupon bonds
d) have lower duration than smaller-coupon
bonds
e) Both (b) and (d)

97.

A client with a large, well-diversified


common stock portfolio expresses concern
about a possible market decline. However,
he/she does not want to incur the cost of
selling a part of their holdings nor the risk
of mistiming the market. A possible strategy
for him/her would be
a)
b)
c)
d)
e)

98.

Buy an index call option.


Sell an index call option.
Buy an index put option.
Sell an index put option.
He cannot protect against the decline
with these options.

The Performance Fund had returns of 19%


over the evaluation period and the
benchmark portfolio yielded a return of 17%
over the same period. Over the evaluation
period, the standard deviation of returns
from the fund was 23% and the standard
deviation of returns from the benchmark
portfolio was 21%. Assuming a risk-free rate
of return of 8%, which one of the following
is the calculation of the Sharpe index of
performance for the fund over the evaluation
period?
a)
b)
c)
d)
e)

Workbook

.3913
.4286
.4783
.5238
.5870

99.

The standard deviation of the returns of a


portfolio of securities will be ____________
the weighted average of the standard
deviation of returns of the individual
component securities.
a)
b)
c)
d)

Equal to.
Less than.
Greater than.
Less than or equal to (depending upon
the correlation between securities).
e) Less than, equal to, or greater than
(depending upon the correlat ion
between securities).
100. According to fundamental analysis, which
phrase best defines the intrinsic value of a
share of common stock?
a) The par value of the common stock.
b) The book value of the common stock.
c) The liquidating value of the firm on a per
share basis.
d) The stock's current price in an inefficient
market.
e) The discounted value of all future
dividends.
101. A call option with a strike price of 110 is
selling for 3 when the market price of the
underlying stock is 108. The intrinsic value
of the call is:
a)
b)
c)
d)
e)

0.
1.
2.
3.
(2).

102. In contrast to the Capital Asset Pricing


Model, the Arbitrage Pricing Theory (APT):
a) Is usually a multi-factor model.
b) Is primarily using by arbitrageurs to profit
from imperfections in security markets.
c) Assumes a market portfolio.
d) Is a useful technical indicator.
103. With the same rupee investment, which of
the following strategies can cause the
investor to experience the greatest loss?
a)
b)
c)
d)
e)

Investment Planning

Selling a naked put option.


Selling a naked call option.
Writing a covered call.
Buying a call option.
Buying the underlying security.

13

104. If the market risk premium were to increase,


the value of common stock (everything else
being equal) would:
a) Not change because this does not affect
stock values.
b) Increase in order to compensate the
investor for increased risk.
c) Increase due to higher risk-free rates.
d) Decrease in order to compensate the
investor for increased risk
e) Decrease due to lower risk-free rates.
105. The current annual dividend of ABC
Corporation is Rs.2.00 per share. Five
years ago the dividend was Rs.1.36 per
share. The firm expects dividends to grow
in the future at the same compound annual
rate as they grew during the past five years.
The required rate of return on the firm's
common stock is 12%. The expected return
on the market portfolio is 14%. What is the
value of a share of common stock of ABC
Corporation using the constant dividend
growth model? (Round to the nearest rupee)
a)
b)
c)
d)
e)

Rs.11.
Rs.17.
Rs.25.
Rs.36.
Rs.54.

Standard deviation.
Variance.
Correlation coefficient.
Coefficient of variation.
Beta.

a)
b)
c)
d)
e)

5; 1.
1; 3.
1; 4.
1; 5.
2; 5.

14

1, 2, and 3 only.
1 and 3 only.
2 and 3 only.
4 only.
1, 3, and 4 only.

108. Which of the following are non-diversifiable


risks?
1
2
3
4
5
6

Business risk.
Management risk.
Company or industry risk.
Market risk.
Interest rate risk.
Purchasing power risk.

a)
b)
c)
d)
e)

4,
1,
5,
1,
4,

5, and 6 only.
2, and 3 only.
6, and 2 only.
3, and 4 only.
and 6 only.

1
2
3
4

Finance foreign exports.


Eliminate currency risk.
Sell U.S. securities in overseas markets.
Trade foreign securities in U.S. markets.

a)
b)
c)
d)
e)

1 and 3 only.
1 and 4 only.
2 and 4 only.
4 only.
1, 2, and 4 only.

110. Which combination of t he f ollowing


statements about bond swaps is true?

107. A Rs.1,000 bond originally issued at par


maturing in exactly 10 years bears a coupon
rate of 8% compounded annually and a
market price of Rs.1147.20. The indenture
agreement provides that the bond may be
called after five years at Rs. 1,050. Which
of the following statements is/are true?
1 The yield to maturity is 6%.
2 The yield to call is 5.45%.

a)
b)
c)
d)
e)

109. American depository receipts (ADRs) are


used to:

106. In computing portfolio performance, the


Sharpe index uses _____ while the Treynor
index uses _____ for the risk measure.
1
2
3
4
5

3 The bond is currently selling at a


premium, indicating that market interest
rates have fallen since the issue date.
4 The yield to maturity is less than the yield
to call.

1 A substitution swap is designed to take


advantage of a perceived yield
differential between bonds that are
similar with respect to coupons, ratings,
maturities, and industry.
2 Rate anticipation swaps are based on
forecast s of general interest rate
changes.
3 The yield pickup swap is designed to
change the cash flow of the portfolio by
exchanging similar bonds that have
different coupon rates.

Investment Planning

Workbook

4 The tax swap is made in order to


substitute capital gains for current yield.
a)
b)
c)
d)
e)

1, 2, and 3 only.
1 and 3 only.
2 and 4 only.
4 only.
1, 2, 3, and 4.

111. Modern "asset allocation" is based upon the


model developed by Harry Markowitz.
Which of the following statements is/are
correctly identified with this model?
1 The risk, return and covariance of assets
are important input variables in creating
portfolios.
2 Negatively correlated assets are
necessary to reduce the risk of portfolios.
3 In creating a portfolio, diversifying across
asset types (e.g., stocks and bonds) is
less effective than diversifying within an
asset type.
4 The efficient frontier is relatively
insensitive to the input variable.
a)
b)
c)
d)
e)

1 and 2 only.
1, 2, and 3 only.
1 only.
2 and 4 only.
1, 2, and 4 only.

112. Which one of the following products is


designed to provide both growth and
income?
a) Fixed premium annuity.
b) Non-participating mortgage real estate
investment trust (REIT).
c) Aggressive growth mutual fund.
d) Convertible bond.
113. Jennifer is optimistic about the long-term
growth of her Widget stock. However, the
stock, currently priced at Rs.58, has made
a sharp advance in the last week and she
wants to lock in a minimum price in case
the shares drop. What might Jennifer do?
a)
b)
c)
d)

Workbook

Buy Rs.55 call options.


Sell Rs.55 call options.
Buy Rs.55 put options.
Sell Rs.55 put options.

114. Company ABC is currently trading at Rs.35


and pays a dividend of Rs.2.30. Analysts
project a dividend growth rate of 4%. Your
client Tom requires a rate of 9% to meet his
stated goal. Tom wants to know if he should
purchase stock in Company ABC.
a) Yes, the stock is undervalued.
b) No, the stock is overvalued.
c) No, the required rate is higher than the
projected growth rate.
d) Yes, the required rate is higher than the
expected rate.
e) No, the required rate is lower than the
expected rate.
115. If the client needs to accumulate wealth but
is risk-averse, which of the following is the
most crucial action the planner needs to take
to have the client achieve the goal of wealth
accumulation? Advise investing the client's
current assets.
a) In the products which will bring the highest
return to the client regardless of risk.
b) In products that produce high income for
the client because fixed income products
are generally safe.
c) In diversified mutual funds because of
the protection which diversity provides.
d) After determining the client's risk tolerance.
e) In 100% cash equivalents in the portfolio
because most soft ware programs
recommend this safe approach.
116. Which of the following would result in the
largest increase in the price of a diversified
common stock mutual fund?
a)
b)
c)
d)

Unexpected inflation.
Expected dividend increases.
Unexpected corporate earnings growth.
Expected increase in the prime interest
rate.

117. In analyzing the position of a portfolio in terms


of risk/return on the capital market line (CML),
superior performance exists if the fund's
position is the CML, inferior performance
exists if the fund's position is the CML, and
equilibrium position exists if it is the CML.
a)
b)
c)
d)
e)

Investment Planning

Above; on; below.


Above; below; on.
Below; on; above.
Below; above; on.
On; above; below.

15

118. Which one of the following best describes


a debenture?
a) A long-term corporate promissory note.
b) An investment in the debt of another
corporate party.
c) A long-term corporate debt obligation
with a claim against securities rather
than against physical assets.
d) A corporate debt obligation that allows
the holder to repurchase the security at
specified dates before maturity.
e) Unsecured corporate debt.
119. A client has a cash need at the end of seven
years. Which of the following investments
might initially immunize the portfolio?
1 A 9-year maturity coupon bond.
2 A 7-year maturity coupon Treasury note.
3 A series of Treasury bills.
a)
b)
c)
d)
e)

3 Lowers average cost per share over a


period of time (assuming share price
fluctuations).
4 Invests the same rupee amount each
month to protect the investment from
loss of capital.
a)
b)
c)
d)
e)

1 and 2.
1 and 3.
2 and 3.
2 and 4.
1, 2, 3, and 4.

Directions for questions 122 and 123: Refer to


the caselet below and answer the questions
Smith invests in a limited partnership that requires
an outlay of Rs.9,200 today. At the end of years 1
through 5, he will receive the after-tax cash flows
shown below. The partnership will be liquidated
at the end of the fifth year. Smith is in the 28% tax
bracket.

1, 2, and 3.
1 only.
2 and 3.
2 only.
1 and 2.

120. Which combination of the following


statements about investment risk is correct?
1 Beta is a measure of systematic, nondiversifiable risk.
2 Rational investors will form portfolios and
eliminate systematic risk.
3 Rational investors will form portfolios and
eliminate unsystematic risk.
4 Systematic risk is the relevant risk for a
well-diversified portfolio.
5 Beta captures all the risk inherent in an
individual security.
a)
b)
c)
d)
e)

a)
b)
c)
d)
e)

17.41%.
19.20%.
24.18%.
28.00%.
33.58%.

123. Which of the following statements is/are


correct?

1, 2, and 5.
1, 3, and 4.
2 and 5.
2, 3, and 4.
2 and 5.

121. Which combination of t he f ollowing


statements is true regarding the investment
strategy known as "Rupee-cost averaging"?
1 Invests the same Rupee amount each
month over a period of time.
2 Purchases the same number of shares
each month over a period of time.

16

122. The after-tax IRR of this investment is:

1 The IRR is the discount rate, which


equates the present value of an
investment's expected costs to the
present value of the expected cash
inflows.
2 The IRR is 24.18% and the present value
of the investment's expected cash flows
is Rs.9,200.
3 The IRR is 24.18%. For Smith to actually
realize this rat e of return, the
investment's cash flows will have to be
reinvested at the IRR.

Investment Planning

Workbook

4 If the cost of capital for this investment


is 9%, the investment should be rejected
because its net present value will be
negative.
a)
b)
c)
d)
e)

2 and 4.
2 and 3.
1 only.
1, 2, and 3.
1 and 4.

Directions for questions 124: Refer to the


caselet below and answer the question.
The tax bracket and holdings of your client are as
follows:
l

Corporate tax bracket = 33%

d) Bond A would have a greater percent


change in price than Bond C because it
has a shorter duration.
e) The percent change in the price of Bonds
A and C is equal since it is not affected
by duration.
126. The following set of newly issued debt
instruments was purchased for a portfolio:
Treasury bond, Z ero-coupon bond,
Corporate bond, Municipal bond.
The respective maturities of these
investments are approximately equivalent.
Which one of the investments in the
preceeding set would be subject to the least
amount of price volatility if interest rates were
to change quickly?
a)
b)
c)
d)

Treasury bond.
Zero-coupon bond.
Corporate bond.
Municipal bond.

127. Which of the following best describes the


investment characteristics of a high-quality
long-term municipal bond?

*There have been no capital gains distributions.


124. During the 12 months from June 30th, last
year, through June 30th, this year, the
portfolio earned, in annual yield and beforetax appreciation, respectively:
a)
b)
c)
d)

5.5% and
5.5% and
6.6% and
6.6% and

17.5%.
21.3%.
17.5%.
21.3%.

a) Bond C, having a longer duration than


Bond A, would have a larger percent
increase in price than Bond A.
b) The percent change in price of a bond is
independent of the duration of a bond.
c) It is not possible to determine the percent
change in price of Bond A versus Bond
C because the duration of Bond C is not
given.

High inflation risk; low default risk.


Low inflation risk; high market risk.
Low inflation risk; low default risk.
High inflation risk; high market risk.

128. Jasmine has a large paper profit in her


Amalgamated Corporation shares, currently
at Rs.46. She is happy with the stock but
realizes that a good thing cannot go on
forever. If she is willing to sell at 50, what
strategy could you recommend to her?
a)
b)
c)
d)

125. Assuming that the duration of Bond A is 1.94


years, which of the following statements
about the effect of a 1% decline in interest
rates is true?

Workbook

a)
b)
c)
d)

Buy Rs.50 call options.


Sell Rs.50 call options.
Buy Rs.50 put options.
Sell Rs.50 put options.

129. "Stock prices adjust rapidly to the release


of all new public information." This
statement is an expression of which one of
the following ideas?
a) Random walk hypothesis.
b) Arbitrage pricing theory.
c) Semi-strong form of the efficient market
hypothesis.
d) Technical analysis.

Investment Planning

17

130. Assuming that the current market yield for


similar risk bonds is 8%, determine the
discounted present value of a Rs.1,000 bond
with a 7.5% coupon rate, which pays interest
semiannually and matures in 17.5 years.
a)
b)
c)
d)
e)

Rs.504.68.
Rs.539.78.
Rs.953.34.
Rs.968.96.
Rs.1,653.26.

1 Purchasing power risk.


2 Interest rate risk.
3 Prepayment risk.
a)
b)
c)
d)

131. Given the following diversified mutual fund


performance data, which fund had the best
risk-adjusted performance if the risk-free
rate of return is 5.7%?

a) Fund B because the annual return is


highest.
b) Fund A because the standard deviation
is lowest.
c) Fund C because the Sharpe ratio is
lowest.
d) Fund D because the Treynor ratio is
highest.
e) Fund A because the Treynor ratio is
lowest.
132. To immunize a bond portfolio over a specific
investment horizon, an investor would do
which of the following?
a) Match the maturity of each bond to the
investment horizon.
b) Match the duration of each bond to the
investment horizon.
c) Match the average weighted maturity of
the portfolio to the investment horizon.
d) Match the average weighted duration of
the bond portfolio to the investment
horizon.

18

133. Mortgage-backed securities may contain


which of the following risks?

2 only.
1 and 2 only.
1 and 3 only.
1, 2, and 3.

134. You receive a phone call from an individual


you have not spoken with previously. The
caller is excited, just having heard that a
new mutual fund is positioned to deliver
large gains in the coming year. The caller
wishes to purchase shares of the fund
through you. Keeping in mind stages of
the overall personal financial planning
process, which of the following questions
that addresses the first two stages of the
financial planning process should you ask
the caller?
1
2
3
4

What are your goals for this investment?


What other investments do you have?
What is your date of birth?
Do you want your dividends reinvested?

a)
b)
c)
d)

1 and 3 only.
2 and 4 only.
1, 2, and 3 only.
1, 2, and 4 only.

135. The duration of a bond is a function of its:


1
2
3
4

Current price.
Time to maturity.
Yield to maturity.
Coupon rate.

a)
b)
c)
d)
e)

1 and 3 only.
2 and 3 only.
2 and 4 only.
1, 2, and 3 only.
1, 2, 3, and 4.

Investment Planning

Workbook

136. Investment A costs Rs.10,000,000 and


offers a single cash inflow of Rs.13,000,000
aft er 1 year. Invest ment B costs
Rs. 1,000,000 and will be worth
Rs.2,000,000 at the end of the year. The
appropriate discount rate or required rate
of return is 10% compounded annually.
Match the investment(s) listed below with
the corresponding financial information
provided.
a)
b)
c)
d)

Investment A.
Investment B.
None of them.
Either A or B.

Rs.28.57.
Rs.38.50.
Rs.40.04.
Rs.41.60.

138. A client has a growth objective but requires


a large percentage of the return to be tax
efficient. Which of the following products
would be most appropriate for this client?
a)
b)
c)
d)

Nonleveraged equipment leasing.


Balanced mutual fund.
Preferred stock mutual fund.
Stock index fund.

139. Rani invested Rs.10,000 in a fixed deposit.


If the interest is compounded monthly at an
annual rate of 4%, what would be the
amount that Rani would receive in five years
time? (use 2 decimal places)
a)
b)
c)
d)

Rs.12,158.65
Rs.12,209.97
Rs.12,188.65
Rs.12.187.65

140. Rekha has borrowed Rs.2,500 at 2%


annual interest rate compounded quarterly.
What is the amount she has to repay after
five years?
a)
b)
c)
d)

Workbook

a)
b)
c)
d)

Rs.12561.26
Rs.12562.11
Rs.12562.26
Rs.12564.26

142. Chetan borrowed Rs.5,000 five years ago.


Assuming that he has to repay Rs.6,500 now,
how much interest rate was he charged?

137. The Zeta Corporation's current dividend is


Rs.3.85. If future dividends are expected to
grow at 4% forever, which of the following
amounts should Zeta stock sell for if the
required rate of return on the stock is 14%?
a)
b)
c)
d)

141. Avni wants to accumulate Rs.15,000 in


three years time for a one-month USA trip
upon her graduation. Assuming she can get
6% annual return compounded semiannually from her investment, how much
must she invest today to achieve her goal?

a)
b)
c)
d)

5.28
5.39
5.93
5.82

143. A bond of face value of Rs. 1000 has a


coupon of 7.5% and is compounded
quarterly duration 4 yrs. similar bond in
market yield 8% what is PV of the bond?
a)
b)
c)
d)

Rs.982.03
Rs.983.03
Rs.984.03
Rs.985.03

144. Mona invests Rs. 21000 for 3 yrs @ 6 %p.a.


compounded annually for first 1 yr and
quarterly for next 2 years. What would be
her maturity benefit?
a)
b)
c)
d)

Rs.25076.35
Rs.25075.72
Rs.18951.71
Rs.20277.68

145. A bond of face value of Rs. 1000 has a


coupon of 7.5% and is compounded semi
annually duration 4.5 yrs. similar bond in
market yield 8% what is PV of the bond?
a)
b)
c)
d)

Rs.981.41
Rs.982.42
Rs.983.25
Rs.980.25

Rs.2,762.24
Rs.2,763.24
Rs.2,769.24
Rs.2,768.28

Investment Planning

19

146. Mahesh invests Rs. 18000 for 4 yrs @ 8 %


p.a. compounded quarterly for first 2 yrs and
annually for next 2 years. What would be
his maturity benefit?
a)
b)
c)
d)

Rs.24599.22
Rs.24566.33
Rs.25501.33
Rs.26502.23

147. A bond of face value of Rs. 1000 has a


coupon of 8.5% and is compounded
annually, duration 12 yrs., similar bond in
market yield 9% what is PV of the bond?
a)
b)
c)
d)

Rs.963.20
Rs.964.20
Rs.964.80
Rs.965.80

148. Mary invests Rs. 15000 for 3 yrs @ 8 %


p.a. compounded annually for first 1 yr and
half yearly for next 2 years. What would be
her maturity benefit?
a)
b)
c)
d)

Rs.18524.21
Rs.18951.71
Rs.19850.71
Rs.19849.21

149. Mr. Singh buys 200 convertible debentures


of TISCO at Rs. 200 each. 50% of the value
of these debentures is converted into one
share of Rs. 80 each after 4 years. Mr. Singh
exercises his options after 4 yrs and
receives 100 shares. Compute cost of
acquisition of each share.
a)
b)
c)
d)

Rs.200
Rs.250
Rs.275
Rs.300

150. Meena invests Rs. 16000 for 4 yrs @ 6 %


p.a. compounded annually for first 1 yr and
quarterly for next 3 years. What would be
her maturity benefit?
a)
b)
c)
d)

20

151. Mr. Satish buys 300 convertible debentures


of ABB at Rs. 300 each. 50% of the value
of these debentures is converted into one
share of Rs.80 each after 5 years. Mr. Singh
exercises his options after 5 yrs and
receives 120 shares. Compute cost of
acquisition of each share.
a)
b)
c)
d)

Rs.350
Rs.360
Rs.375
Rs.380

152. A bond of face value of Rs. 1000 has a


coupon of 7.5% and is compounded semi
annually, duration 17.5 yrs., similar bond in
market yield 8% what is PV of the bond?
a)
b)
c)
d)

Rs.956.34
Rs.953.34
Rs.1074.43
Rs.987.60

153. Miss Savi buys 250 convertible debentures


of Reliance Ind. at Rs. 300 each. 60% of
the value of these debentures is converted
into one share of Rs. 80 each after 7 years.
Miss. Savi exercises his options after 7 yrs
and receives 100 shares. Compute cost of
acquisition of each share.
a)
b)
c)
d)

Rs.400
Rs.420
Rs.425
Rs.450

154. A bond of face value of Rs. 1000 has a


coupon of 6.5% and is compounded semi
annually, duration 13.5 yrs. similar bond in
market yield 7% what is PV of the bond?
a)
b)
c)
d)

Rs.957.79
Rs.985.79
Rs.956.79
Rs.958.79

Rs.20277.68
Rs.20278.22
Rs.22285.25
Rs.20275.78

Investment Planning

Workbook

155. Miss Seema buys 250 convertible


debentures of Asian hotels at Rs. 300 each.
60% of the value of these debentures is
converted into one share of Rs. 100 each
after 7 years. Miss. Seema exercises his
options after 7 yrs and receives 150 shares.
Compute cost of acquisition of each share.
a)
b)
c)
d)

a)
b)
c)
d)

Rs.225
Rs.250
Rs.275
Rs.300

156. A bond yield 10% annually with interest paid


semi- annually. Duration is 3 years. The
yield on bond is 12%. Find current price.
a)
b)
c)
d)

Rs.1000.83
Rs.1050.43
Rs.986.36
Rs.950.83

Rs.200
Rs.225
Rs.250
Rs.275

Rs.912.98
Rs.912.38
Rs.913.28
Rs.914.88

Workbook

Rs.31706.49
Rs.30706.49
Rs.30705.29
Rs.30704.29

162. Reshma invests Rs. 5000 @8%


compounded monthly .What will be the
value of his investment after 4 years?
a)
b)
c)
d)

Rs.6877.33
Rs.6876.33
Rs.6878.33
Rs.6876.88

a) Book value based on net tangible assets.


b) Liquidation value based on the proceeds
of liquidation of the company.
c) Present value of all the dividends to be
received from holding that share.
d) Apply the P / E ratio to expected earnings
per share.
164. If a bond is selling at a premium _______.

159. Ravi invests Rs.15000 @12% compounded


monthly .What will be the value of his
investment after 6 years?
a)
b)
c)
d)

Rs.978.62
Rs.978.42
Rs.978.52
Rs.979.72

163. The best method of valuing a share is:

158. A bond of face value of Rs.1000 has a


coupon of 7.5% and is compounded
quarterly, duration 16 yrs, similar bond in
market yield 8.5% what is PV of the bond?
a)
b)
c)
d)

Rs.375
Rs.380
Rs.385
Rs.390

161. A bond of face value of Rs. 1000 has a


coupon of 7.5% and is compounded semi
annually duration 5 yrs, similar bond in
market yield 8% what is PV of the bond?
a)
b)
c)
d)

157. Mrs. Shikha buys 250 convertible


debentures of ACC. at Rs. 300 each. 60%
of the value of these debentures is
converted into one share of Rs. 50 each
after 8 years. Mrs. Shikha exercises his
options after 8 yrs and receives 200 shares.
Compute cost of acquisition of each share.
a)
b)
c)
d)

160. Mr. Sushil buys 300 convertible debentures


of Telco at Rs. 250 each. 40% of the value
of these debentures is converted into one
share of Rs. 50 each after 6 years. Mr. Sushil
exercises his options after 6 yrs and receives
80 shares. Compute cost of acquisition of
each share.

a) It is an attractive investment
b) Its realised compound yield will be less
than the yield to maturity
c) Its coupon rate is below market rate
d) Its current yield is lower than the coupon rate
165. Which of the following is NO T a
characteristic of a balance fund?
a) It provides both growth and income
objectives
b) It is less risky than growth funds
c) It is more risky than income funds
d) It must invest in both equity and bonds
in equal amount

Investment Planning

21

166. Purohit has just bought a house for


Rs.8,00,000. He wants to sell it in 20 years
time for Rs. 20,00,000. His friend who is a
real estate agent estimates that the house
will appreciate in value by 6% for the first 8
years, 5% for the next 8 years and 4.50%
for the last four years. What will be the value
of the house after 20 years and will Purohit
be able to realise his desired value?
a)
b)
c)
d)

Rs.21,85,489, Yes
Rs.22,46,552, Yes
Rs.19,54,413, No
Rs.17,45,387, No

167. Disclosure statements to prospective clients


include all of the following except:
a) Performance record of other clients.
b) The method of remuneration, fees and
commissions.
c) Access to internal and external complaint
handling mechanism.
d) Disclosure of any conflict of interest.
168. Which of the following costs best describes
the cost of foregone income that results from
making an economic decision to use funds
to purchase a piece of equipment?
a)
b)
c)
d)

Cost of Capital
Fixed Cost
Marginal Cost
Opportunity Cost

169. A mutual fund that invests in Indian Equities,


foreign equities, Indian Corporate Bonds,
Indian Government Gilts is subject to the
following risks?
1.
2.
3.
4.

Business Risk,
Default Risk,
Systematic Risk,
Interest Rate Risk.

a)
b)
c)
d)

1 & 3 only
1,3 & 4 only
3 & 4 only
1,2,3 & 4

a)
b)
c)
d)

1 only
1 & 2 only
2 & 3 only
1,2 & 3

171. If a new issue was offered to the public at


15 times earnings but the market was
pricing similar shares at 19 times, this would
be ____________.
a)
b)
c)
d)

Appalling proposition to the investor


The investor cannot take a position
An example of low gearing
Bargain not to be missed

172. Mr. X buys 50 TISCO October Rs. 350 call


options for Rs. 15. The current share price
is Rs. 345. The break-even share price,
ignoring transaction costs is Rs.________.
a)
b)
c)
d)

350
360
365
None of the above

173. Which of the following investor complaints


relating to the following Capital Market
issues will not be entertained by SEBI?
a) A company declaring no dividend on
equity for the fourth consecutive year.
b) A company has declared dividend but
not paid the same after six months of
declaration.
c) A company not paying the redemption
proceeds on debentures issued by the
company, one year after maturity date.
d) None of the above cases.
174. In ranking portfolio performance, which
measure of risk does the Treynor Index
use?

170. Which of the following statements


concerning technical stock market indicators
is/are correct?
1. The stock market is considered strong
when the volume of the market is
increasing in a rising market.

22

2. The market's direction will change when


the percent of odd-lot short sales
significantly increases or decreases.
3. Prices crossing the moving average line
would be an indication of the change in
the market.

a)
b)
c)
d)

Investment Planning

standard deviation
variance
beta
alpha

Workbook

175. A portfolio manager can hedge a share


portfolio by_______.
a)
b)
c)
d)

Buying call options


Selling call options
Buying index options
Selling put options

176. The trust deed lays down the terms and


conditions under which the unit holders money
is to be invested. It specifically details:
a)
b)
c)
d)

The unit trust scheme


The types of authorized investments
All fees and charges
All of the above

177. Beta is a measure of _________


a)
b)
c)
d)

Geometric average return


Holding period return
Systematic risk
Unsystematic risk

178. If the current share price is S and the set


exercise price is X, the intrinsic value of the
call option is_______.
a)
b)
c)
d)

Max (O, S-X)


Max (O, X-S)
Min (O, S-X)
Min (O, X-S)

179. The CAPM is a model that:


a) Determines the geometric return of a
security.
b) Determines time-weighted return
c) Explain return in terms of risk.
d) Explains systematic risk
180. GE is an AAA rated issuer of Corporate
Bonds in the International Debt markets.
The issue price of a typical GE corporate
bond is affected by all the following EXCEPT
the _________.
a) Face value, coupon rate, and maturity
of the bond.
b) Firms required return on debt.
c) Percentage of debt in the firm's capital
structure.
d) Required return on the firm's competitors'
bonds.

Workbook

181. Public Issue through the book building


process is better than I.P.O at fixed price
because_______.
a) High fixed price will result in under
subscription leading to loss to the investor.
b) It helps the issuer to ascertain the exact
price at which the investor is willing to
subscribe.
c) Low fixed price will result in over
subscription leading to loss to the issuer.
d) All of the above
182. A client purchased a mutual fund with a
Rs.10, 000 lump-sum amount four years
ago. During the four years, Rs.4, 000 of
dividends was reinvested. Today the
shares are valued at Rs.20,000 (including
any shares purchased with dividends). If
the client sells shares equal to Rs.13,000,
which statement (s) is/are correct?
1. The taxable gain can be based on an
average cost per share.
2. The client can choose which shares to
sell, thereby controlling the taxable gain.
3. To minimize the taxable gain today; the
client would sell shares with the higher
cost basis.
4. The client will not have a gain as long as
he/she sells less than what he/she
invested.
a)
b)
c)
d)

1, 2, and 3 only.
1 and 3 only.
2 and 4 only.
only.

183. Which combination of t he f ollowing


statements about investment risk is correct?
1. Beta is a measure of systematic, nondiversifiable risk.
2. Rational investors will form portfolios and
eliminate systematic risk.
3. Rational investors will form portfolios and
eliminate unsystematic risk.
4. Systematic risk is the relevant risk for a
well-diversified portfolio.
5. Beta captures all the risk inherent in an
individual security.
a)
b)
c)
d)
e)

Investment Planning

1, 2, and 5.
1, 3, and 4.
2 and 5.
2, 3, and 4.
2 and 5.

23

184. Which of the following asset allocations


would you recommend to a 60 year old
retiree who depends on his investments for
monthly income?
a) Fixed Deposits:
Equities: 10%
b) Fixed Deposits:
Equities: 30%
c) Fixed Deposits:
Equities: 40%
d) Fixed Deposits:
Equities: 50%

60% Properties: 30%


40% Properties: 30%
20% Properties: 40%
10% Properties: 40%

185. Rank the following funds in the order of


increasing risk:
I
II
III
IV

Growth Fund,
Balanced Fund,
Bond Fund,
Small Cap Fund

a)
b)
c)
d)

III, IV, II, I


III, II, IV, I
III, II, I, IV
III, IV, I, II

Sponsor
AMC
Trustee
Custodian

Derivatives market
Real Estate market
Fixed Income Market
Stockmarket

24

Fixed Deposits
Mutual Funds
Equity Shares
Real Estate

a)
b)
c)
d)

Customised
Novated
Standardised
Without counterparty risk

193. __________ equity funds are appropriate


for an Indian investor with a flair for global
diversification
Diversified
Sector
International
Global

194. Your liquid fund investment is treated as a


____________ asset
a)
b)
c)
d)

Cash Flow Planning


Investment Planning
Will Planning
Insurance planning

189. Studies show that the Indian middle class


has an excess of ______ as investments.
a)
b)
c)
d)

191. _________ funds have the highest expense


ratio

a)
b)
c)
d)

188. Implied volatility is a term commonly used


in the __________
a)
b)
c)
d)

Service tax
Securities Transaction Tax
Brokerage
Depository Participant Charges

a) Confers a right but no obligation on the


buyer
b) Confers an obligation but no right to the
buyer
c) Enables an investor to take a leveraged
position
d) Protects the buyer against downside risk

187. Capital allocation forms a critical part of


___________
a)
b)
c)
d)

a)
b)
c)
d)

192. Purchasing a Futures Contract __________

186. The Fund Manager for a particular scheme


is an employee of the _______
a)
b)
c)
d)

190. Only those equity transactions where


______ is paid are eligible for zero long term
capital gains tax

Intangible
Investment
Near-Cash
None of the above

195. Which of the following factors will result in an


increase in the duration of corporate bond.
I. an increase in the number of years to
maturity.
II. a decrease in the coupon rate
III. change from annual to semiannual
coupon payment.
IV. change from annual coupon to zero
coupon bond
a)
b)
c)
d)

Investment Planning

I, II & III only


I, II and IV only
I, III & IV only
II, III & IV only

Workbook

196. A lump sum investment option in a mutual


fund will be most warding in _________
a)
b)
c)
d)

A secular uptrend
A secular downtrend
Volatile markets
A sideways market

197. Tata Power is raising funds through a bond


issuance to fund a new power plant at
Noida, UP. They are issuing Two Year
maturity, Zero-coupon bond with face value
of Rs 1000 and yield of 4%. What price
would you pay for this Tata Power Zerocoupon bond today?
a)
b)
c)
d)

145.64
156.22
168.59
172.56

199. The risk free return of Security A is 8%. In


addition to it, you expect that the return on
market would be 14%. The expected return
of Security A with beta of 0.70 is ________.
a)
b)
c)
d)

12.2%.
15.4%.
17.8%.
18.2%.

200. The debt - asset ratio is a useful tool to


primarily measure ______________
a)
b)
c)
d)

Workbook

Liquidity
Solvency
Savings potential
Debt servicing capacity

a)
b)
c)
d)

0.94
1.07
1.31
1.91

202. The following are statements made


concerning contracts of insurance. Identify
the statement/s that is/are correct.

Rs. 920.00.
Rs. 924.56.
Rs. 925.95.
Rs. 960.00.

198. Equal amount of investment is made in


portfolio consisting of securities X and Y.
Standard deviation of X is 12. 43%.;
Standard deviation of Y is 16. 54%.;
Correlation coef ficient is 0.82.; The
interactive risk of the portfolio, measured by
covariance is __________
a)
b)
c)
d)

201. Portfolio A had a return of 12% in the


previous year, while the market had an
average return of 10%. The standard
deviation of the portfolio was calculated to
be 20%, while the standard deviation of the
market was 15% over the same time period.
If the correlation between the portfolio and
the market is 0.8, what is the Beta of the
portfolio A?

I. For life insurance contracts,


misstatement of the insured's age
constitutes avoidable misrepresentation.
II. An innocent misrepresentation by an
applicant for insurance constitutes fraud.
a)
b)
c)
d)

I only
II only
I and II
Neither I nor II

203. Which of the following statements


concerning the movement in f oreign
exchange rates are true, assuming that all
other factors remain unchanged?
I. The exchange rate will appreciate with
an increase in domestic interest rates
II. Increasing international reserves leads
to a stronger exchange rate
III. Strengthening the exchange rate leads
to a temporary increase in the
competitiveness of exports
IV. Lower domestic inflation leads to a
weakened exchange rate
a)
b)
c)
d)

Investment Planning

I & II only
II & III only
III & IV only
I & IV only

25

204. A corporation proposes to issue a 7-year


bond with a coupon rate of 8.0%. The
prevailing yield to maturity of bonds with
similar risk and term is 10.0%.The bond will
sell at ____________ to its face value.
a)
b)
c)
d)

a premium
a discount
par
a predetermined price

205. Calculate the net price earnings ratio (PER)


of the stock of Company A, with the following
information:
Price = Rs.5.00
Profit before tax = Rs.75.0 million
Profit after tax = Rs.54.0 milllion
Paid up Capital = Rs.100 million at par value
of Rs.0.50 per share
a)
b)
c)
d)

6.7
9.3
13.3
18.5

weak form of market efficiency


semi-strong form of market efficiency
strong form of market efficiency
total market efficiency

207. Which of the following have a negative


impact on stock prices?
I. increase in risk premiums
II. increase in dividend growth rate
III. increase in the discount rate
IV. increase in interest rates
a)
b)
c)
d)

I II & III only


I II & IV only
I, III & IV only
II, III & IV only

a)
b)
c)
d)

Stock B will move up by 12%


Stock B will move down by 12%
Stock B will move up by 9%
Stock B will move down by 9.0%

210. Which of the following statements regarding


investment risk are true?
I. Beta captures all the inherent risks in an
individual security
II. Unsystematic risk is reduced in a
portfolio because securities are not
perfectly correlated
III. As Beta increases, the expected return
also increases
IV. Rational investors will form portfolios to
eliminate systematic risk
a)
b)
c)
d)

I & II only
II & III only
III & IV only
I & IV only

211. Which of the following are among the


advantages of money market funds?
I. Provides current income
II. Provides safety of principal
III. Notice is required for withdrawal
IV. Provides capital gains
a)
b)
c)
d)

26

25.00
11.00
9.75
9.50

209. Stock A and stock B are positively correlated


with a correlation coefficient of 0.75. When
stock A moves up by 12%, how will stock B
perform?
a)
b)
c)
d)

206. Mr A's stock market portfolio consistently


outperformed the BSE Sensex based on tips
he received from a friend working in a
corporate finance department. Which of the
following is his success a violation of?
a)
b)
c)
d)

208. What is the expected return of an


investment with the following distribution of
possible outcomes?

Investment Planning

I only
I & II only
I, II and III only
All of the above

Workbook

212. Mr A is 30 years of age and intends to retire


when he has Rs.400,000. He believes that
he can save Rs.10,000 at the end of each
year. Being risk adverse, he places his
funds in fixed deposits earning only 5.0%
pa. His savings at this point is Rs.20,000.
At what age will he be able to retire?
a)
b)
c)
d)

53 years
51 years
68 years
70 years

213. What is the net present value (NPV) of a


Rs.8.0 million investment with the following
cash flows:

Assume a discount rate of 10% pa for all


years.
a)
b)
c)
d)

Rs.2.23 million
Rs.3.8 million
Rs.5.0 million
Rs.10.5 million

214. Mr A places his fixed deposit for one year


with interest compounded annually at
maturity. Mr B places his fixed deposit with
another bank with interest compounded
quarterly. Assuming that the deposit amount
is Rs.100,000, and using the same annual
rate of 6%, what is the difference in future
value after one year?
a)
b)
c)
d)

Rs.130.43
Rs.132.54
Rs.134.22
Rs.136.36

215. Which of the following assumptions support


the use of technical analysis?
a) Future performance should be reflective
of past performance
b) The values of market indices and stock
prices are determined based on supply
and demand
c) Stock prices move in trends that would
persist over long periods
d) All the above

Workbook

216. Of the four pairs of portfolios, which pair


provides the highest level of diversification?
a) Portfolio 1 & 2: with
coefficient of + 0.92
b) Portfolio 3 & 4: with
coefficient of + 0.37
c) Portfolio 5 & 6: with
coefficient of 0
d) Portfolio 7 & 8: with
coefficient of -0.78

a correlation
a correlation
a correlation
a correlation

217. The profit earned by a Mutual Fund will be


directly proportional to __________
a)
b)
c)
d)

the level of the BSE Sensex


the assets under management
its operating expenses
the sponsor's net worth

218. Anil Kumar has two Mortgage Loan options


before him. The interest rate, and other
conditions are the same for both, except that
one has a repayment term of 15 years and
the other has a repayment term of 30 years.
Anil wants to evaluate the EMIs for both
terms. All other conditions being the same,
repaying a loan in 15 years instead of 30
would require EMIs that are________.
a) Half the size of the 30-year loan
payments.
b) Less than twice as large as the 30 year
loan payments.
c) More than twice as large as the 30 year
loan payments.
d) Twice as large as the 30 year loan
payments.
219. You are running a Dividend Yield Fund for a
leading Mutual Fund House. The most recent
dividend of All Is Fine Business Services
common stock was Rs 2.35. The dividends
are expected to grow at 4 percent indefinitely.
If you are looking at a 12 percent return, how
much will you be willing to pay for one share
of All Is Fine Business Services?
a)
b)
c)
d)

Investment Planning

Rs. 24.79.
Rs. 29.38.
Rs. 30.55.
Rs. 32.45.

27

220. You are an owner of an apartment complex with


300 units, each of which can fetch Rs. 1000
p.m. as rentals. The apartment complex has an
average occupancy rate of 75%. The expenses
for maintaining, up keeping the apartment
comes to around Rs. 10 Lakh p.a. Based on
the concept of capitalized earning approach and
assuming that you require a capitalization rate
of 10%, how much is the complex worth now?
a)
b)
c)
d)

Rs. 1.50 crore


Rs. 1.70 crore
Rs. 2.00 crore
None of the above.

221. Which of the following are true about


Securities A and B?
Given the following information about
securities A and B:
Historical Returns for Securities:

a)
b)
c)
d)

1 and 3
1 and 4
2 and 3
2 and 4

222. Given the capitalization rate of the following


link houses in the BKC area, how much should
your client pay for the link house in BKC 4?

28

Rs.
Rs.
Rs.
Rs.

Rs.114
Rs.105
Rs.100
Rs. 62

225. An investor invested Rs.4000 in each of


Funds A & B with details as provided below.
Both funds subsequently appreciated by
18% based on their NAV.Which of the
following statements is/are true?

I. The investor obtained 3,921.57 units in


Fund A.
II. The investor obtained 6,896.55 units in
Fund B.
III. The investor would have achieved a higher
return by investing in Fund A alone
IV. The investor would have achieved a higher
return by investing in Fund B Alone
a)
b)
c)
d)

a)
b)
c)
d)

8.10%
7.69%
7.00%
6.05%

224. Using the Growth Dividend Model, calculate


the price of the stock of Company A, with
the following information:
EPS = Rs.10 per share
Current Dividend = Rs.8 per share
Dividend growth rate = 5.0%
Risk free rate = 6.0%
Company A risk premium = 7.0%
a)
b)
c)
d)

1) A is more risky because it has a higher


standard deviation.
2) B is less risky because it has a lower
standard deviation.
3) A has a lower risk-adjusted return.
4) B has a higher risk-adjusted return.
a)
b)
c)
d)

223. Calculate the yield to maturity of a bond with


the following parameters:
Face Value : Rs.1000.00
Market Price : Rs.1040.00
Coupon Rate (paid annually) : 7.0%
Remaining Term to Maturity : 5 years

I & II only
I, II & III only
I, II & IV only
III only

306,657
303,121
299,041
293,786

Investment Planning

Workbook

226. What is the duration of a zero coupon bond


with yield to maturity of 6% maturing in 6
years time?
a)
b)
c)
d)

4.35
5.34
6.00
6.35

227. Calculate the return of an investor who


bought units of Fund A on 31/12/06 and sold
the units on 31/12/07 with the information
provided below:

233. Classifying an investment as a long term


investment depends primarily on:
a) the length of time the investor expects
to hold the investment.
b) the amount of the investment.
c) whether a liquid market exists for selling
the investment.
234. A period when an economy is experiencing
substantial growth and a declining jobless
rate is called _____________.
a)
b)
c)
d)

a)
b)
c)
d)

235. In India Futures contracts in _______ may


be settled by delivery.

0.9%
4.4%
11.3%
17.0%

228. The most appropriate investment for a


highly risk averse investor aged 57 is _____
a)
b)
c)
d)

an index fund based on the Nifty.


a diversified equity fund.
a Maharashtra State Government Bond.
a SBI Fixed Deposit.

229. In industry life cycle, revenue, margin and profit


are expected to peak in the following order:
a)
b)
c)
d)

Revenue, margin, profit


Margin, profit, revenue
Margin, revenue, profit
Profit, margin, revenue

230. The broadest measure of inflation is:


a) Consumer Price Index
b) Wholesale Price Index
c) Both (a) and (b)
231. NPV is calculated in the case of a series of
_________ cash flows.
a)
b)
c)
d)

Zero
Single
Uneven
Even

232. The Present Value of a sum of money as


the Discounting Rate ____________.
a)
b)
c)
d)
e)

Workbook

Remains same, Increases


Decreases, Decreases
Increases, Increases
Increases, Decreases
Data Insufficient

Stagflation
Deflation
Depression
Boom

a) Commodities
b) Stocks
c) Stock Index
236. Money has time value. It derives this value due
to existence of several conditions. Which one
of the following is not one of the conditions
contributing to the existence of this value?
a) Possibility of increase in tax rates over
time.
b) Ability to buy/ rent assets generating
revenue
c) Cost of foregoing present consumptions
d) None of the above.
237. You have term deposits of Rs. 4,00,000 with
a bank. In order to meet sudden
requirements for liquidity and short-term
credit, you are applying for an overdraft
facility with the bank. What is the rate of
interest you will pay on this facility?
a) The bank will apply a flat rate of interest
on the amount of overdraft allowed to
actually utilize.
b) The bank will apply a flat rate of interest
on the amount of overdraft allowed to you.
c) The bank will apply rate of interest linked
to the term deposit rate, on the amount
of overdraft utilized.
d) The bank will apply rate of interest linked
to the term deposit rate, on the average
amount of overdraft remaining

Investment Planning

29

238. The Nifty has doubled since the last time


you advised your client to reduce his equity
exposure. The client is annoyed. What might
be the most appropriate action to take
immediately?
a) Apologize for wrongly forecasting the
market
b) Change his asset allocation by
increasing his equity exposure
c) Help the client understand the logic of
his asset allocation
d) Rebalance his asset allocation by
reducing equity investments
239. A professional indemnity policy protects the
insured from risk arising out of ________.
a) Intentional misconduct
b) Misrepresentation of professional
competence
c) Negligence
d) Undisclosed conflict of interest
240. Protector International is a financial services
firm that specializes in investment advisory
services. In its brochure for Financial
Planning services, it may state ________.
a) It can offer superior investment returns
on customer portfolios and talk of the
arrangements to offer advice in other
areas
b) It has the competence to take care of all
Financial advisory requirements of the
customer
c) Its competence in investment advisory
services and the arrangements to offer
advice in other areas
d) Its Financial Planning services are the
best available in the market in light of its
investment advisory capabilities and
arrangements to offer advice in other
areas
241. Which of the following is a concurrent
indicator of the phase of the business cycle?
a)
b)
c)
d)

30

Wholesale price Index


Index of Industrial production
Labor costs and capacity utilization
Order levels in the manufacturing sector

242. The effective interest rate earned per rupee


_________ as the periods of compounding
increase.
a)
b)
c)
d)

Increases
Decreases
Remains same
Decreases for some time and then
increases
e) Data insufficient
243. What is the main difference between the
personal Financial Planning needs of the
employed and the self-employed?
a)
b)
c)
d)

Attitude to risk/risk appetite


Need to fund children's education
Need to fund retirement
The extent of any employer-provided
pension benefits

244. Immunization protects bondholders from


which of the following risk/s:
1) Interest rate risk
2) Reinvestment rate risk
3) Maturity risk
a)
b)
c)
d)

1 only
2 only
1 & 2 only
1, 2 & 3

245. The economy is going through a phase of


expansion and growth. Industrial production
and profitability are high. Your client has a
portfolio that is heavily invested in bonds.
Which of the following fears of the client is
well founded?
a) Higher rates of growth will increase
demand for funds and interest rates will
firm up, leading to fall in bond prices.
b) Higher rates of growth will require higher
imports and expenses. The government
deficits will go up.
c) The central bank will try to reduce rates
to make funding of business cheaper and
reduce costs.
d) The currency will become convertible
and i nt erest rat es wi ll rise as a
consequence.

Investment Planning

Workbook

246. Ram is a Financial Planner in a large firm.


His wife has some large investments in the
shares of a few companies. Ram is required
to offer views on almost all of these holdings
to clients. Under the Code of Ethics and
Rules of Professional Conduct _______

251. If your investment goal is simply to match


the market, you should buy a(n)

a) Ram must disclose the fact to his


client(s) so as to make them aware of
any potential conflict of interest
b) Ram has to disclose these holdings only
to his employers, if required by the firm's
internal compliance rules
c) Ram need not follow any code of ethics
and rules of professional conduct.
d) Ram will not violate the Code and the
Rules if he does not disclose his wife's
holdings

252. You are evaluating a fund. What activity would


you typically not undertake in this effort?

247. How are financing costs included in NPV


and IRR calculations?
a) By including them in the interest
payments.
b) By considering the interest rate in the
setting of the discount rate
c) As a tax deduction
d) By including them in the earnings
248. The term "Efficient Frontier" is contained
in________.
a) Technical Analysis
b) Modern Portfolio Theory
c) Value Investing Theory
249. A major difference between load and noload funds is :
a) Marketability: no-load funds can be
traded more readily.
b) Acquisition cost: load funds cost more
than their NAVs.
c) Performance: load funds do better.
250. The Reliance fund trades on the NSE. Its
recent price is Rs.10, but its NAV is Rs.12.
We know then :
a) The fund is closed-end, selling at a
discount.
b) The fund is open-end, selling at a
premium.
c) The fund is closed-end, selling at a
premium.

Workbook

a) Growth Fund
b) Money Market Fund.
c) Index Fund.

a) Calculate or find the fund's rate of return.


b) Calculate the fund's NAV.
c) Find the fund's turnover ratio and
administrative expenses-to-assets ratio.
253. The Cholamandalam Fund's rate of return
was 9%, while the market return was 15%.
Cholamandalam's beta was 0.5.
a) Cholamandalam's management outperformed the market on a risk-adjusted
basis.
b) Cholamandalam's RAROR was 11.5%.
c) Cholamandalam's management under
performed the market on a risk-adjusted
basis.
254. You are considering taking a passenger with
you when you go home over Christmas
break. She lives 100 kms out of your way,
and the total trip is 500 kms. She has offered
Rs.500 for the service. You estimate the
total cost of the trip at Rs.3000. You should
a) Reject the offer since 20% (1000/5000)
of Rs.3000 is greater than Rs.500.
b) Accept the offer if the opportunity costs
of the trip is greater than Rs.2500.
c) Accept the offers if marginal costs
associated with 100 kms are less than
Rs.500.
255. Liquidity ratio of 2.0 tells us that the family
has :
a) Rs.2 in liquid assets for each Rs.1 in total
liabilities.
b) Rs.2 in liquid assets for each Rs.1 in
current liabilities.
c) Rs.2 in liquid assets for each Rs.1 of total
expenses.
256. In India, Preference shares may be issued
for a maximum number of ____ years.
a)
b)
c)
d)

Investment Planning

12
15
10
20

31

257. Determination of Residential status for the


tax purpose is applicable on:
i.
ii.
iii.
iv.

Individual
Firm
Company
HUF

a)
b)
c)
d)

i,ii,
i,ii, iii
i only
i, ii, iii, iv

258. A growth-oriented non-divided paying


share is bought for Rs. 250 and sold for
Rs. 450 after 5 years, the compound
annual growth rate is:
a)
b)
c)
d)

14.86%
12.47 %
11.50%
10.71%

259. The call option strike price on a share


is Rs. 500 and the current share price
is Rs. 550. The call option premium is
Rs. 60. The time value of the option is:
a)
b)
c)
d)

60
10
30
15

260. Mr. A deposits Rs. 10,000 in his own PPF


account and same amount in his wifes
account. How much maximum amount can
he deposit in his nephews name?
a)
b)
c)
d)

Rs. 20,000
Nil
Rs. 70,000
Rs. 60,000

261. A Rs.100 par value bond having 10 %


coupon rate will mature after 7 years. Find
the value of the bond if the discount rate is
8 %.
a)
b)
c)
d)

32

Rs.109.85
Rs.111.41
Rs.108.75
Rs.110.41

262. Consider a portfolio of two investments viz.


A & B. The sum total of volatility of A
and B respectively, represented by
standard deviation of t he two
investments, will be equal to the volatility
of the portfolio as a whole if _________.
a) A and B have a correlation of Zero
b) A and B have a correlation of 1
c) The portfolio is equally divided between
A and B
d) The return on the portfolio is equal to the
sum of returns of A and B
263. Which of the following is a correct
interpretation of the Rules of Conduct
pertaining to the Ethic of Confidentiality?
a) A Member must when requested by the
client, provide to a person authorized by
the client, all original documents
prepared or received by the Member in
undertaking the advisory task
b) A Member owes t o the Member's
partners or co-owners a responsibility to
act in good faith (expectations of
confidentiality) only while in business
together, not thereafter
c) The Member shall maintain the same
standards of confidentiality to employers
as to clients
d) Under no circumstance, will any Member
divulge any information or knowledge
regarding the FPSB India or its members
that they may know or be exposed to
264. Mr. Rajan's investment portfolio comprises
Rs.2 lakh in equity, Rs.5 lakh in debt
and Rs.1 lakh in his bank current account.
Over one year the returns on equity and
debt are 5% and 12%. At the end of the
year to maintain his current asset allocation,
he needs to _____________.
a) do nothing.
b) to move Rs, 10000/- from equity and Rs.
60000/- from debt to cash.
c) to move Rs.7500/- to equity from debt
and Rs. 8750/-to cash from debt
d) to invest Rs. 70000/- in debt and equity.

Investment Planning

Workbook

265. A 10 year 8.0% bond (Face ValueRs.1000, interest payable semi-annually)


maturing 6 years from today is available
at a yield to maturity of 6.0%. It is likely to
be priced at ____________.
a)
b)
c)
d)

Rs.
Rs.
Rs.
Rs.

1100
1149
1168
1498

266. In India, Mutual Funds have recently moved


to the concept of _______ AUM calculation.

a)
b)
c)
d)

6.40%
3.2%.
6.5%.
7.2%.

270. Which of the following is a tort of


negligence?

a) Monthly average
b) Month end
c) Fortnightly average
267. Raykar is an accomplished Financial Planner
and is also an expert on derivatives and high
yielding bonds. He understands client
requirements well and is able to come up with
appropriate portfolio restructuring ideas for
clients. He believes in quickly moving clients
from one investment to another through a dynamic
process of research and recommendations. What
according to the Rules relating to the Code
of Ethics is the most applicable in this case?
a) He does not violate the Rules if he
explains to the client the reasons and is
able to show that the moves are
appropriate to the client
b) He does not violate the Rules since he
conducts and has access to research
and advises on products relevant to
clients based on an understanding of
their requirements
c) He does not violate the Rules since he
is an acknowledged expert and knows
what is best for his clients
d) He violates the Rules as it amounts to
active churning of client portfolios
268. Mrs. & Mr. Arora are aged 55 and 58 years
respectively. Both expect to work till they turn
65. Their only goal is to fund their retirement.
Which of the following is likely to be an
appropriate asset allocation strategy for them?
a) 10% sectoral equity, 20% diversified
equity, 30% long-term debt, and 40%
medium term debt
b) 20% sectoral equity, 60% diversified
equity, 20% long-term debt
c) 30% sectoral equity, 30% diversified
equity, 40% cash/ liquid investments.
d) 80% long-term debt, 20% medium term
debt
Workbook

269. ABC Ltd. is willing to prepay your


Cumulative Fixed Deposit with them,
without any penalty and with all the
accumulated interest (compounded half
yearly). You had invested Rs. 4000 with
them 3.5 years back. If they are giving you
back Rs. 4985, what is the annualized rate
of interest you have earned?

a) Mr. Joy was playing golf. He swings a


new golf club on the fairway and the
head of the club flies off, and hit another
golfer who was standing 20 feet away.
b) Mr. Vishal takes medication that he
knows makes him drowsy and then
proceeds to drive. He gets into an
accident injuring the passengers in
another car.
c) Mrs. Jaya locks Ms. Rani in a room to
prevent her from leaving the building
d) Mrs. Priti experienced a sudden surge
of chest pain while driving, which causes
her to lose control of her car and hit
another car.
271. Any possible occurrence which may have
a negative financial implication, can be
plotted on a graph with X axis measuring
the frequency (low-high) and Y axis
measuring the financial impact (low-high).
You can view the classification in four
quadrants.
Quadrant I - Low frequency, Low Impact
Quadrant II - Low frequency, High Impact
Quadrant III - High frequency, High Impact
Quadrant IV - High frequency, Low Impact
It would not be practical to purchase
insurance for events falling in _________.
a)
b)
c)
d)

Investment Planning

Quadrant I & IV
Quadrant I, II & IV
Quadrant I, III & IV
Quadrant III

33

272. Bond A has a 6% annual coupon and is


due in 2 years. Its value in today's market
is Rs. 900. Bond B has a 10% annual
coupon and is due in 4 years. It is priced
to yield 12%. Bond C is a 9% zero-coupon
bond priced to yield 11% in 8 years. The
yield to maturity of Bond A is closest to:
a)
b)
c)
d)

9.90%.
10.40%.
10.90%.
11.90%.

Bond C is a 9% zero-coupon bond priced


to yield 11% in 8 years. Assuming that the
duration of Bond A is 1.94 years, which of
the following statements about the effect of
a 1% decline in interest rates is true?
a) Bond C, having a longer duration than
Bond A, would have a larger percent
increase
b) The percent change in price of a bond is
independent of the duration of a bond.
c) It is not possible to determine the percent
change in price of Bond A versus Bond C
d) Bond A would have a greater percent
change in price than Bond C because it
has a shorter duration.
274. The new Senior Citizens Savings Bond
Scheme offers ________ % Interest.
8.5
9
9.25
8.75

275. The maximum amount that can be invested


in Public Provident Fund is Rs. _______.
a)
b)
c)
d)

70000
60000
80000
90000

276. ___________ is regulated by the Reserve


Bank of India.
A. Bank Deposit Rates;
B. Bank Lending Rates;
C. Certificate of Deposit Rates
a)
b)
c)
d)
34

A
B
C
None

a)
b)
c)
d)

Equity Shares
Debt Mutual Funds
Property
Gold

278. The deduction from Gross Total Income


available u/s 80DDB is Rs.___________.

273. Bond A has a 6% annual coupon and is due


in 2 years. Its value in today's market is
Rs.900. Bond B has a 10% annual coupon
and is due in 4 years. It is priced to yield 12%.

a)
b)
c)
d)

277. The recent Union Budget exempts ______


from Long Term Capital Gains tax subject
to certain conditions.

a) 40000
b) 45000
c) 50000
279. _________ is / are governed by SEBI.
a)
b)
c)
d)

Mutual Funds
Stock Brokers
Portfolio Managers
All of the above

280. A person can be qualified as an Associate


Financial Planner after he/she passes
modules of the CFP Certification Course.
a)
b)
c)
d)

6
1
3
5

281. The first step of the financial planning


process is ___________.
a)
b)
c)
d)

Evaluating the various Alternatives


Data gathering and goal setting
Establishing the Client Planner relationship
Plan Review

282. How many years will it take for a sum of


Rs.10000 to double if the rate of return is
9% p.a.?
a)
b)
c)
d)
e)

9.5
8.5
10
9
8

283. If the post tax rate of return on an investment


is 8% and the inflation rate is 5% the real
rate of return is _____.
a)
b)
c)
d)
e)

Investment Planning

3.5%
3.0%
2.86%
-3.0%
2.74

Workbook

284. The key reference rate for inter-bank overnight


borrowings in Mumbai is known as
a)
b)
c)
d)

MIBOR
MIBID
LIBOR
LIBID

285. This relationship between the trustees & the


unit holders of a mutual fund shareholders
is called a __________ relationship
a)
b)
c)
d)

Contractual
Fiduciary
Moral obligatory
None of the above

286. Selling the wrong type of policies in order


to earn higher commissions will render an
insurance agent to be held liable ________.
a)
b)
c)
d)

under law of tort


for professional misconduct
for professional negligence
under the SEBI Act

287. In portfolio management, a constant ratio


plan is one that _____________.
a) Restores asset holdings at the end of a
period to target weights.
b) Buys different kinds of securities in a
fixed proportion.
c) Pays out dividends in a constant ratio to
the portfolio's asset values.
288. As interest rates rise bond prices _____ and
the longer are their maturities the _______
is the price change.
a) Rise; lesser
b) Fall; lesser
c) Fall; greater
289. Which item below is not a met hod
recommended for acquiring securities?
a) Dividend reinvestment plans
b) Rupee cost averaging
c) Averaging alpha and beta weights
290. KM, Inc. has a beta of 1.5. The risk-free rate is
5%, and the market risk premium is 8%. Thus,
a) KM has a required return of 6.5% and
an expected return of 8%.
b) KM has a required return of 15.5%.
c) KM has a required return of 9.5%.

Workbook

291. NASDAQ is _____________


a) A section of the NYSE where technology
stocks are traded.
b) The trading symbol for a racetrack
company listed on the NYSE.
c) The NASDAQ (acronym of National
Association of Securities Dealers
Automated Quotations) is an American
stock exchange.
292. An equity mutual fund has an NAV of Rs. 15
on January 1, 2006. You invest Rs. 15000 on
that day. You sell these units at an NAV of
Rs. 25 on January 15, 2007. The entry load
is 2.25% while the exit load is 0.50%. You are
in the 30% tax bracket. Your pre-tax return
on the same is _____%
a)
b)
c)
d)

66.66
62.18
46.66
42.18

293. The thumb rule states that we should


choose an investment with a _______ IRR.
a.
b.
c.
d.

Lower
Higher
Equal
Data insufficient

294. Seema and Arun are co-applicants of a


mortgaged house. They are on the verge
of a divorce. The Housing Finance
Company will ___________
a) not interfere as long as the EMIs are
being paid on time.
b) repossess the house after divorce
c) insist on the house being transferred to
one of them.
d) mediate reconciliation between the
couple.
e) Increase the interest rate in order to
compensate for the increased risk.
295. Domestic GOI bond holders (holding them
up to Maturity) have to deal with _____ risk.
a)
b)
c)
d)
e)

Investment Planning

Volatility
Default
Inflation
Price
Currency

35

296. Refinancing is ______________.


a) Borrowing at lower cost in order to pay
off higher cost debt
b) Repaying debt by selling off assets
c) Lending at a higher rate of interest
d) Securitizing your receivables
e) None of the above
297. ________________ Asset Allocation is not
a text book Asset Allocation Model.
a)
b)
c)
d)
e)

Tactical
Discretionary
Strategic
All of the above
None of the above

298. For a nominal interest rate of 6% payable


monthly, quarterly, and semi-annually, the
effective rates respectively would be _____.
a)
b)
c)
d)

6.04, 6.02, 6.01


6.16, 6.13, 6.09
6.10, 6.07, 6.03
6.11, 6.08, 6.06

299. A 10 year 9 % Bond (Face Value of Rs.100,


interest payable annually) maturing 3 years
from today is available at a YTM of 5.8%.
Therefore the current price is ___________.
a)
b)
c)
d)

5. Collect, analyze, and evaluate client


data.
a)
b)
c)
d)
e)

1, 3, 5, 4, 2.
5, 1, 3, 2, 4.
1, 5, 4, 3, 2.
1, 5, 3, 4, 2.
1, 4, 5, 3, 2.

302. A model that describes the relationship


between risk and expected return and that
is used in the pricing of risky securities is
better known as _________
a)
b)
c)
d)

Efficient Market hypothesis


CAPM
Security Market Line
Beta Model

For questions 303 and 304: Refer to the data


below.
The stock of Maxi Limited performs
relatively well to other stocks during
recessionary periods. The stock of Taxi
Limited, on the other hand, does well during
growth periods. Both the stocks are
currently selling for Rs.100 per share. You
assess the rupee return (dividend plus price)
of these stocks for the next year as follows:

Rs.152.50
Rs.154.10
Rs.108.59
Rs.152.00

300. A one-time principal repayment for a bond


is known as a ____________ repayment.
a)
b)
c)
d)

Laddered
Step-Up
Step-down
Bullet

301. Arrange the following financial planning


functions into the logical order in which a
professional financial planner performs
these functions.
1. Interview clients, identify preliminary
goals.
2. Monitor financial plans.
3. Prepare financial plan.
4. Implement financial strategies, plans,
and products.

36

Calculate the standard deviation of investing:


303. Rs.1, 000 in the equity stock of Maxi Limited
a)
b)
c)
d)

Investment Planning

120.5
124.6
116.6
123.4

Workbook

304. Rs.1, 000 in the equity stock of Taxi Limited


a)
b)
c)
d)

301.4
322.6
291.4
296.8

305. Rs.500 each in the equity stock of Maxi


Limited and Taxi Limited.
a)
b)
c)
d)

90.6
104.6
78.4
89.3

Rs.6802
Rs.6870
Rs.6878
Rs.6925

307. Sudha invests Rs.5000 per year (at the


beginning of each year) for 5 years @ 5%
p.a. in a bank deposit. She then withdraws
the accumulated sum over a period of 3
equal annual installments. What is the value
of the deposit at the end of 5 years and the
quantum of withdrawal each year?
a)
b)
c)
d)

a) Rs.117591
b) Rs.119487
c) Rs.118274
For questions 310 and 311 Refer to the data
below:

306. Sanjeev invests Rs.5000 in a Bank


Deposit today @ 8% p.a compounded
monthly. He hopes that this investment
will enable him to fund his college education
(estimated t o cost Rs.9000) which
commences after 4 years. What will be the
value of this investment in four years?
a)
b)
c)
d)

309. Neeta wants to accumulate Rs.1, 50,000 in


three years time for a one month trip to the
USA. Assuming she can get an 8% annual
return on her investments, compounded
quarterly, how much must she invest today
in order to achieve her goal?

Rs.28505, Rs.9954
Rs.29010, Rs.10656
Rs.29568, Rs.11054
Rs.28804, Rs.10042

308. Amar wants to purchase a Car 5 years


from now. His investments are currently
worth Rs.50,000/- and he intends to
contribute Rs.10,000/- at the beginning of
every six months period to fund his purchase.
Assuming that the annual investment rate
of return is 8% compounded semi annually,
what will be the value of the investment in
five years time?

John has estimated that the following will


be his outgoings over the next few years:
End of Year Cash Outflow
Year 1 Rs.10000
Year 2 Rs.15000
Year 3 Rs.12000
Year 4 Rs.13500
Year 5 Rs.11000
310. If John wants to cater to these cash
outflows, how much should he have today,
assuming an annual rate of return of 5%?
a)
b)
c)
d)

Rs.54126
Rs.53221
Rs.52483
Rs.50483

311. Mr. John has purchased 100 convertible


debentures of Essar Oil on 1/1/94 at Rs.500
each. 40% of the value of the debentures is
convertible into one share of Rs.50 each
after seven years. Mr. John exercised his
option on 1/4/2001 and received 100 shares.
Compute the cost of acquisition of these
shares.
a)
b)
c)
d)

Rs.200
Rs.205
Rs.195
Rs.185

a) Rs.1,98,875
b) Rs.1,95,555
c) Rs.1,97,240

Workbook

Investment Planning

37

312. The maxim "buy term and invest the


balance" may not be a feasible proposition
for many investors for the following reasons
EXCEPT for:
a) it may be difficult to achieve sufficient
diversification in the invested assets
b) it may be difficult to achieve a suitable
investment portfolio with the desired risk
reward relationship
c) it may be possible to consistently
outperform the investment returns
earned by an established life office
d) that the insurer offers capital guarantees
on cash values
313. An individual may prefer investment-linked
assurance to conventional assurance
policies for the following reasons EXCEPT
a) he has some direction over the
investment of his premiums
b) he is informed of the expenses charged
for the services provided
c) he is attracted by the guaranteed
surrender values offered under these
contracts
d) he prefers the switching facilities
available under such contracts
314. Which of the following does not constitute
a valid charge on the premiums paid for an
investment-linked life assurance policy?
a)
b)
c)
d)

bid-offer spread
front-end charges
recurrent fund related charges
surrender penalty

315. You are an independent financial planner


and are faced with the problem of suggesting
a suitable insurer for a client who is in very
good health. Assuming that a temporary
assurance policy is needed for this client,
you would recommend an insurer _______.
a)
b)
c)
d)

38

a stringent underwriting policy


a lax underwriting policy
very few causes of death excluded
extensive causes of death excluded

316. Identify the class of assurance for which an


insurer, in a costing exercise, must provide
for future anticipated improvements in
mortality
a)
b)
c)
d)

investment-linked endowment assurance


conventional endowment assurance
group life assurance
life annuities

317. A type of risk with high frequency but low


severity is probably best handled by:
a) Self-insurance
b) Insurance
c) Avoidance
d) Transfer
318. 'Consideration' under the law is a return
promise to:
1.
2.
3.
4.
a)
b)
c)
d)

Do certain things
Abstain from doing certain things
Forbear some acts
Accept an offer
[1], [2], and [3] only
[1], [2], and [4] only
[1], [3], and [4] only
[2], [3], and [4] only

319. A reversionary bonus is:


a) Any profit distributed to holders of life
policies who cash in those policies
b) Generally profit that will be distributed
on the same terms as the sum assured
c) A distribution of income to shareholders
d) Profit distributed to holders of unit-linked
policies
For questions 320 to 322 use the information
below :
A client of yours, who is now aged 35 years, states
that his retirement objective is to provide for himself
at retirement at age 55, annual incomes of 2/3 of
his last drawn salary. He further states that he
expects this income for 20 years after retirement.
His current annual salary is Rs. 60,000 and this
escalates at 6% per annum throughout his career,
with the annual escalations occurring on his
birthdays. Assume an 8% per annum interest rate
throughout.

Investment Planning

Workbook

320. Calculate his last drawn annual salary.


a)
b)
c)
d)

Rs.128,285
Rs.181,536
Rs.192,428
Rs.132,000

321. Calculate the present value of the retirement


corpus.
a)
b)
c)
d)

Rs.225,800
Rs.250,900
Rs.13,60,287
Rs.305,780

a)
b)
c)
d)

Rs.
Rs.
Rs.
Rs.

676774
776774
931095
609870

325. The correlation co-efficient of two perfectly


correlated assets is _____

322. Calculate the level annual amount that he


must deposit until age 54, assuming that the
first is due now, to fund for the stated
retirement income.
a)
b)
c)
d)

324. Mohan invested Rs. 420000 for 7 years @


7% where it was compounded annually for
the first 5 years and quarterly for the last
2 years. What did he receive on maturity?

a)
b)
c)
d)

326. A zero coupon bond will have zero _____ risk


a)
b)
c)
d)

Rs.22,800
Rs.25,965
Rs.28,890
Rs.30,389

For question 323 : Refer to the data below.


Participating ordinary life policies with a sum
assured of Rs. 10,000 are issued by two
life assurance companies, Office 1 and
Office 2, to lives aged 35 with the following
cost data:

+1
-1
+0.5
-0.5

Interest Rate risk


Inflation risk
Reinvestment risk
Default risk

327. Shyam has a one year old daughter. He


wants to save for her higher education which
shall begin at age 21. Which investment
vehicle has the best potential for him to
attain his goals?
a)
b)
c)
d)

Derivative Instruments
GOI Bonds
Bank Fixed Deposits
Equity Mutual Funds

328. _________ is a form of Secured Loan


a)
b)
c)
d)
323. Based on the above information and
assuming a 6% per annum interest rate, the
annual surrender cost index for each
Rs.1,000 sum assured at the end of 20
years
a)
b)
c)
d)

Workbook

For Office 1 is greater than Office 2


For Office 2 is greater than Office 1
For Office 1 is equal to Office 2
Is not determinable

Loans against shares


Credit Card Debt
Holiday Loans
All of the above

329. Beta is a term commonly used in the _____


a)
b)
c)
d)

Art market
Real Estate market
Fixed Income Market
Stockmarket

330. Bank Fixed Deposits eligible for 80 C


deduction must have a minimum tenure of
______ years
a)
b)
c)
d)

Investment Planning

3
4
5
6

39

331. __________ bonds are eligible investments


u/s 54 EC
a)
b)
c)
d)

NHAI
NABARD
REC
Both (a) and (c)

332. _____________ are generally the preserve


of the Wealthy class as compared to the
Middle class.
a)
b)
c)
d)

Fixed Deposits
Mutual Funds
Equity Shares
Collectibles

333. The Mumbai Stock Exchange is a _______.


a)
b)
c)
d)

Corporate entity
Partnership Firm
Association of Persons
None of the above

334. _____ funds have the lowest expense ratio.


a)
b)
c)
d)

Low P/E Ratios


Low Book Values
Low Dividend Yields
Low Margin of safety

338. Narrow money does not comprise of _____.


a) Currency with the public
b) Demand deposits with the banking
system
c) Other deposits with the RBI
d) Time Deposits with banks

40

340. Sharpe ratio determines return per unit of


__________.
a)
b)
c)
d)

Diversifiable Risk
Non-diversifiable risk
Both of the above
None of the above

341. Treynor ratio uses _______ as the divisor.


a)
b)
c)
d)

Beta
Standard Deviation
Variance
Either (b) or (c)

a)
b)
c)
d)

Alpha
Beta
Gamma
Vega

a)
b)
c)
d)

Listed
Unlisted
Both (a) and (b)
Foreign

344. An important characteristic of gilt securities


are that they ___________.

Upward sloping
Downward sloping
Flat
Humped

337. Value investing involves purchasing stocks


with __________
a)
b)
c)
d)

Discount rate
Repurchase Rate
Reverse Repurchase Rate
Prime Lending Rate

343. SEBI is the Regulator for _____ companies.

Personal Use
Investment
Near-Cash
None of the above

336. The yield curve is usually __________.


a)
b)
c)
d)

a)
b)
c)
d)

342. Jensen's ratio helps in determining ______.

Equity
Gilt
Index
Liquid

335. Your residence is treated as a _____ asset.


a)
b)
c)
d)

339. The rate at which commercial banks and


other lending facilities can borrow short-term
funds from the central bank is called the
________.

a) are issued by the Central Government.


b) all have terms to maturity that are 270
days are less.
c) all tend to have large amounts of
purchasing power risk.
d) are issued by corporates
345. One standard deviation means that an
observation will fall between the upper and
lower bound ______ % of the time.
a)
b)
c)
d)

Investment Planning

50.25%
68.30%
75.5%
95%

Workbook

346. The statistical measure of covariance is


known as ___________
a)
b)
c)
d)

Standard Deviation
Correlation coefficient.
Variance.
Level of confidence.

a)
b)
c)
d)

347. ________ is a measure of systematic risk


a)
b)
c)
d)

Correlation coefficient
Beta
Alpha
Sharpe Ratio

348. Profits earned through trading exchange


traded options are treated as _____ gains.
a)
b)
c)
d)

353. The YTM of a Bond is also known as its


_______.

Short Term Capital


Speculative
Long Term Capital
None of the above

349. The features of collectibles include ______.

Internal Rate of Return


Holding period Return
Current Yield
None of the above

354. Equity Linked Savings Schemes eligible for


Sec. 80 C benefits have a lock-in period of
____ years.
a)
b)
c)
d)

2
3
4
5

355. The thumb rule states that we should


choose an investment with a _______ IRR.
a)
b)
c)
d)

Lower
Higher
Equal
Data insufficient

a) The fact that they can be easily valued.


b) The fact that they are traded on a
Collectible Exchange
c) The fact that they have the potential for
great capital appreciation.
d) The fact that common people can invest
in them.

356. Yield to Call is usually calculated for a


_______.

350. ________ is a widely used metric for


measuring volatility of an investment.

357. The relationship between price and interest


rate is ___________.

a)
b)
c)
d)

Mean
Median
Standard Deviation
Variance

a)
b)
c)
d)

351. The IRR method assumes that all cash flows


are reinvested at __________
a)
b)
c)
d)

The cost of capital


The market rate of return
The prevailing PLR
The rate decreed by the RBI

352. A rating of AAA (ind) by Fitch indicates that


the particular security is free from _______
risk.
a)
b)
c)
d)

Workbook

a)
b)
c)
d)

Debt Instrument with an option


Option free instrument
Derivative instrument
Data insufficient

Linear
Convex
Random
Concave

358. The following is not correct about a Gilt


Mutual Fund.
a) They primarily invest in securities issued
by the Central Government.
b) They primarily invest in longer dated
instruments.
c) They are not subject to Capital Gains
Tax if held for a period of over one year.
d) They are more volatile as compared to
liquid funds.

Interest rate risk


Default risk
Inflation risk
Reinvestment risk

Investment Planning

41

359. When we take into account only negative


deviations from the mean, we term it as
________.
a)
b)
c)
d)

Standard Deviation
Variance
Semi-variance
Mode

360. How many years (approx.) will it take for a


sum of Rs.10000 to quadruple if the rate of
return is 9% p.a.?
a)
b)
c)
d)

Rs.1,10,000
Rs.1,13,451
Rs.1,15,000
Rs.1,11,500

362. Suppose you invest in 4 securities. Company


A has an expected return of 20%. Company
B 10%. Company C 12% and Company D
9%. You have invested Rs. 40,000. What
more information is needed to find out the
return on the portfolio?
a)
b)
c)
d)

Beta of these shares


Proportion of investment
Market Value of the investments
None of the above

363. If Standard deviation is 8, the variance is


equal to ________
a)
b)
c)
d)

56
64
72
80

364. _________ distribution, indicates that an a


particular investment has a probability of
one potentially large loss as against several
potentially small gains.
a)
b)
c)
d)

42

Lognormal
Positively skewed
Negatively skewed
Normal

a)
b)
c)
d)

5.50
6.25
6.67
7.05

366. A bank deposit of Rs. 25000 will earn an


interest of Rs._____ at the end of one year,
if it earns 10% p.a. compounded every
month.
a)
b)
c)
d)

18
16
14
12

361. Geeta invests Rs. 2000 at the beginning of


each month for 48 months. Her rate of return
is 8% p.a. The investment's value at the end
of the said period will amount to _______.
a)
b)
c)
d)

365. The P/E Ratio of the Sensex is 15. The


earnings yield is ______

2599
2617
2745
2799

367. A bondholder buys a bond for Rs.105 and


earns Rs.10 p.a. as interest. His current
yield is ______ %
a)
b)
c)
d)

9.85
9.69
9.52
9.39

368. A bondholder buys a bond maturing in one


year for Rs. 90 and earns Rs. 5 per annum
as interest. His holding period yield is ____ %
a)
b)
c)
d)

15.85
16.67
17.25
18.19

369. John has an investment with an annual


income of Rs.100 and current value of
Rs.6,000. If the value of the market is
expected to rise to Rs.7200 by end of 3 years,
the approximate yield on the investment is
_______ %
a)
b)
c)
d)

7.25
7.58
7.85
8.02

370. ________ losses suffered in stock market


related transactions can be carried forward
for upto _______ years.
a)
b)
c)
d)

Investment Planning

Short Term Capital, 8yrs


Speculative, 8yrs
Long Term Capital, 8yrs
Both (a) and (b).

Workbook

371. Ram is an income seeking who is low on


the risk curve. His Financial Planner
suggests that he invest in a Company Fixed
Deposit rated FB by CRISIL, which is
offering a yield of 300 basis points over the
FD rated FAAA. His advice is ________
a) Appropriate because Ram is able to earn
high interest.
b) Appropriate because Ram is low on the
risk curve.
c) Inappropriate because Ram is low on the
risk curve
d) Inappropriate because Ram can earn
even higher income by investing in a FD
rated FC.
372. Senior Citizens Bonds offer _____% interest
and are eligible for _______ tax deduction
a)
b)
c)
d)

Rs.11.00
Rs.12.50
Rs.10.00
Rs.10.50

374. A bondholder buys a bond maturing in two


years for Rs. 120 and earns Rs. 15 per
annum as interest. His YTM is _____ %
a)
b)
c)
d)

4.00
4.55
4.75
4.95

375. The net asset value (NAV) of a closed end


fund has risen from Rs.52 to Rs.62. The
fund was last quoted at Rs. 40, prior to the
announcement of the increase in the NAV.
If the fund were to trade at the same
discount to NAV, estimate the new price of
the fund?
a)
b)
c)
d)

Workbook

a) 9.1%
b) 8.5%
c) 10.0%
377. During the past five years, the returns of a
stock were as follows:

8.5, 80C
9.0, 80C
9.5, 80C
8.75, Nil

373. A company offers a rights issue of one for


three for Rs.5 each. The present share price
is Rs.13. If the share price does not change
during the time of trading, what is the price
after the rights are taken up?
a)
b)
c)
d)

376. You are thinking of acquiring some shares


of ABC Ltd. The rate of return expectations
are as follows:

Rs.48
Rs.52
Rs.54
Rs.57

Find the followings:


a)
b)
c)
d)
e)

Cumulative wealth index


Arithmetic mean
Geometric mean
Variance
Standard deviation

378. This year M.A. Ltd. will pay a dividend on


its stock of Rs. 6 per share. The following
year the dividend is expected to be the
same, increasing to Rs. 7 the year after.
From that point on, the dividend is expected
to grow at 4% per year indefinitely. Stocks
with similar risk are currently priced to
provide a 10% expected return. What is the
intrinsic value of M.A. Ltd.?
379. H. Pipes has issued a preferred stock that
pays Rs. 12 per share. The dividend is fixed
and the stock has no expiration date. What
is the intrinsic value of H. Pipes preferences
stock, assuming a discount rate of 15%.?
380. ABC Ltd. currently earns Rs. 45 per share
its return on equity is 20% and it retains 50%
of its earning (both figures are expected to
be maintained indefinitely). Stocks of similar
risk are priced to return 15%. What is the
intrinsic value of ABC Ltd's Stock?

Investment Planning

43

381. A non-dividend paying stock has a current


price of Rs. 16. What will be the future's price
if the risk free rate is 9 percent and the maturity
of the futures contract is 1 month?
382. Suppose a stock Index has a current value
of 3500. If the risk-free rate is 8 percent and
the expected yield on the index is 2 percent,
what should be the price of a six months
maturity futures contract ?
383. R.P. Singh owns a Rs. 1000 face-value
bond with three years to maturity. The bond
makes annual interest payments of Rs. 75,
the first to be made one year from today.
The bond is currently priced at Rs. 975.48.
Given an appropriate discount rate of 10%,
should R.P. Singh hold or sell the bond?
384. Poonam recently purchased a bond with a
Rs. 1000 face value, a 10% coupon rate,
and four years to maturity. The bond makes
annual interest payments, the first to be
received one year from today. Ramesh paid
Rs. 1032.40 for the bond.

Calculate the portfolio's expected return and


standard deviation if Manoj invests 20% in
stock A, 50% in stock B and 30% in stock
C. Assume that each security's return is
completely uncorrelated, with the returns of
the other securities.
388. Consider two stocks, P & Q

The returns on the two stocks are perfectly


negatively correlated.
What is the expected return of a portfolio
constructed to derive the standard of
portfolio return to zero?
389. The following information is available:

a) What is the bond's yield-to-maturity?


b) If the bond can be called two years from
now at a price of Rs. 1100, what is its
yield-to-call.
385. A Rs. 10,000 face-value bond with a tenyear term-to-maturity. Currently sells so as
to produce on 8% yield-to-maturity.
Calculate the bond's price if its yield rises
to 10%, if its yield falls to 5%.
386. At the beginning of the year, the Saturn fund's
NAV was Rs. 18.50. At the end of the year its
NAV was Rs. 16.90. At year-end the fund paid
out Rs. 1.25 in income and capital gains. What
was the return to an investor in the Saturn
fund during the year?
387. Manoj Kumar owns three stocks and has
estimated the following joint probability
distribution of returns:

a) What is the covariance between stock A


and B?
b) What is the expected return and risk of
a portfolio in which A and B are weighs
of 0.6 and 0.4
390. Consider the following information for three

The mean risk-free rate was 6 percent.


Calculate the treynor measure, sharp
measures, and Jensen measure for the
three mutual funds and the market index.

44

Investment Planning

Workbook

391. Vipul Ahaju's worth of the portfolio in the


beginning of the year was Rs. 39,000. At
year-end, Vipul received a gift of Rs. 4,000,
which was invested in the portfolio. The
portfolio's value at year-end was Rs. 42,000.
What was the return on the Vipul's portfolio
during the year?

For questions 395 to 397: Refer to the datd


below.
Standard deviation of an asset 2.5%
Market standard deviation

2.0%

Risk-free rate of return

13.0%

Expected return on market


portfolio
Correlation co-efficient of
portfolio with market

392.

395
The mean risk free rate was 10%.
Calculate Treynor measure, Sharpe
measure and Jensen measure.
For questions 393 and 394 Refer to the data
below :
The market portfolio has a historically based
expected return of 0.095 and a standard deviation
of 0.035 during a period when risk-free assets
yielded 0.025. The 0.06 risk premium is thought
to be constant through time. Risk less investments
may now be purchased to yield 0.08.
A security has a standard deviation of 0.07 and a
0.75 correlation with the market portfolio. The
market portfolio is new expected to have a
standard deviation of 0.035.
393. Market's return-risk trade off will be _____.
a)
b)
c)
d)

1
2
0.035
None of above

394. Beta security will be _____.


a)
b)
c)
d)
e)

Workbook

0.07
0.035
1
1.5
None of above

15.0%
0.8

Market sensitive index will be _______.


a)
b)
c)
d)

1.25
0.8
10
1

396. Expected rate of return on the portfolio will


be _______.
a)
b)
c)
d)
e)

13%
15%
14.6%
16%
None of the above

397. If the portfolio beta is 0.5 and risk free return


10%, expected return on the portfolio will
be ________.
a)
b)
c)
d)

10%
15%
12.5%
11.25%

For questions 398 to 401: Refer to the data


below.
Sachin goes to Sanjeev, a CFP, for his capital
market investments. Sachin has regularly been
receiving dividend. His total income falls in the
highest tax slab. He has received bonus shares
and right shares from certain companies.
398. The benefit of indexation is available in
respect of transfer of __________.
a) Short-term capital Assets
b) Long-term Capital assets
c) Both short-term as well as long-ter capital
assets
d) Neither short-term nor long-term capital
assets

Investment Planning

45

399. In case of bonus shares :


a) Entire net sale consideration is taxed as
capital gain
b) Entire net sale consideration is not taxed
as capital gain
c) Capital gain will be reduced by the
proportionate cost of original shares
d) None of above
400. Sachin has renounced the right shares in
favour of his friend Sanjay @ Rs. 5 per
share. If Sachin wished, he could have
purchased the right share @ Rs. 25 per
share.
In respect of renouncement of right share:
a) Sachin is not required to pay capital gain
tax
b) There will be short-term capital gain @
Rs. 5 per share
c) There will be short-term capital gain @
Rs. 20 per share
d) None of above
401. If Sachin opts for indexation benefit in
respect of his listed securities. The resultant
capital gain will be taxed at
a)
b)
c)
d)

Slab rates
20% plus surcharge, if any
10% plus surcharge, if any
Maximum marginal rates

For questions 402 to 404 Refer to the data


below :
Steve Waugh wants to be actively involved in
derivative trading soon after taking retirement from
cricket. He has no knowledge about this field and
has approached you for guidance.
402. Which of the following statement is true?
a) Price of an option is called as Premium
b) A short call is buying a call possibly with
the hope of selling it back later at lower
price
c) A short call is selling a call possibly with
the hope of buying it back later at a
higher price
d) A short call is selling a all possibly with
the hope of buying it back later at a lower
price
e) Both (a) & (d)

46

403. Which of the following statements are true


about future contracts?
a) Forward price are normally higher than
spot prices
b) Usually the position are marked to
market
c) Daily settlement takes places
d) Both (a) and (b)
e) (a), (b) as well as (c)
404. Intrinsic value is:
a) Difference between original and current
share price
b) Difference between original and strike
price
c) Difference between strike and current
share price
d) None of above
For questions 405 to 407 Refer to the data
below :
A bond has a face value of Rs. 1,000 & coupon
rate of 8%. The required rate of return is 6%.
405. What will be the value of bond if the bond is
perpetual?
a)
b)
c)
d)

Rs. 1,333
Rs. 1,000
Rs. 667
None of above

406. If the maturity life of the bond is only 5 years,


value of bond will be ___________.
a)
b)
c)
d)

Rs. 1,084
Rs. 1,000
Rs. 916
None of the above

407. If in above question, the required rate of


return is 10%, the value of bond will be
__________.
a)
b)
c)
d)

Investment Planning

Increased by Rs. 160


Decrease by Rs. 160
There will be no impact
Be Rs. 1,000

Workbook

408. In respect of fixed income securities, which


of the following statement is not true?
a) Current yield includes only the coupon if
the security is sold immediately
b) Current yield includes both coupon and
capital gain/loss if the security is sold
immediately
c) YTM is the return an investor would
receive if the security were held to maturity
d) All are true
e) All are false
For questions 409 to 414 Refer to the data
below :
Parmar is a CFP, who wants to sharpen his skill
about mutual funds as he has taken up a business
in the last two years and has been totally out of
touch of this field. He now wants to again take up
his profession. But before that the wants to test
his knowledge about mutual funds.
409. Open ended funds __________.
a) are available for subscription throughout
the year
b) have a fixed maturity
c) can be bought and sold at NAV
d) all of the above are true
e) only (a) and (c) are true
410. NAV of one unit at a mutual fund is Rs. 11.
The entry load is 4%. The cost to the
investor would be ___________
a)
b)
c)
d)

Rs. 11
Rs. 11.44
Rs. 10.56
Rs. 11.88

a)
b)
c)
d)

High liquidity
High income
Security of investment
All are available

413. Which of the following features are present


in Exchange Traded Funds?
a)
b)
c)
d)
e)

Real Time NAV


Daily/Real Time Portfolio Disclosure
Low cost intra-day trading possible
All of the above
(a) & (c)

414. The analysis of mutual funds should focus


on:
a)
b)
c)
d)
e)

Investment personnel
Investment philosophy & process
Past performance
All of above
(a) & (c)

For question 415 to 417 Refer to the data


below :
Your client has put all his money, so far, in financial
investments. Recently, his interest has arisen in
real estate investments. But he is treating it as
similar to investment in financial investments. You
explained him that real estate, as an investment
is different from investment in f inancial
investments. He wants to know more about real
estate as an investment.
415. Which of the following is true?

411. Which of the following advantages is not


available to a person investing in mutual
funds?
a) control over actual investment
b) selection flexibility to invest or withdraw
funds
c) transparency
d) convenient administration
e) all are available

Workbook

412. Which are following features is not present


in money market mutual funds?

a) Agricultural income is exempt from


income tax
b) Agricultural income is exempt from
wealth tax
c) Real estate investment is illiquid
d) All of above
416. Real estate market in India is ________.
a)
b)
c)
d)

Investment Planning

Highly organized
Disorganized
Is free from Government controls
Offers homogeneous products

47

417. What ails the Indian Housing Industry?


a)
b)
c)
d)

ii)

Lack of clear titles in most cases


High stamp duty rates
Obsolete tenancy and rental control laws
All of the above

For Questions 418 to 420 Refer to the data


below :

a)
b)
c)
d)
iii)

iv)

a)
b)
c)
d)

Only by cheque
Only in cash
Only by draft
Any of the above mode

a)
b)
c)
d)

Six years
Eight years
Six and a half years
Depends on the investor

420. Which of the following is true, relating to


6.5% RBI Saving bonds?
a)
b)
c)
d)
e)

Premature withdrawal is allowed


Tax rebate u/s 88 is available
Interest is tax free
Interest is payable only at maturity
All of above

421. Mrs. Saradha Menon has made investments


in the following five securities on 1.1.2004.
The number of shares bought, purchase
price and the expected price on 31.12.2004
are shown in the table below:

25%
28%
17.5%
33.3%

Which of the following figures is approximately


equals the return on the portfolio?
a)
b)
c)
d)

419. The maturity period of NSC is ________.

25%
28%
18%
30%

What is the return on security E?


a)
b)
c)
d)

Ramesh wants to invest in small saving schemes


as they offer higher interest rates. However, he does
not know about other intricacies of these schemes.
418. NSC can be purchased:

What is the expected rate of return on


security A?

15%
17.25%
18.5%
26.75%

422. Mrs. Ramana has a portfolio comprising


equities, bonds and real esate. The
standard deviation are 0.1689, 0.0716 and
0.0345 respectively. The coorelations are
0.45 for equity and bonds, 0.35 for equity
and real estate, 0.2 for bonds and real
estate.
Find out standard deviation of the portfolio,
if the weights are 25%, 50% and 25%
respectively.
a)
b)
c)
d)

6.5%
3.25%
6.10%
8%

Refer to the question above.


423. Assume that the proportions of investments
are 20% 35% and 40% what is the risk of
the portfolio?
i.
ii.
iii.
iv.

9.5%
9%
8%
6.4%

424. During the last five years, Mr. Saxena


owned securities that had the following
annual rates of return:
i)

What is the weight of Security D in the above


portfolio now?
a)
b)
c)
d)

48

27%
22%
13.5%
8%

Investment Planning

Workbook

Which is the preferable security by each of


the following measure?
i)

v)

a)
b)
c)
d)

Arithmetic mean annual rate of return


a) Security A
b) Security B
c) Both are equally preferable

ii)

Geometric mean annual rate of return


a) Security A
b) Security B
c) Both are equally preferable

What is the correlation coefficient between


stocks and bonds?
+0.34
-0.43
+0.43
-0.1452

426. Assume that you own two securities with


the following expected returns and standard
deviations. The proportion of holding is also
indicated.

425. The following are the assessment of the


probabilities of each of the five economic
scenarios:

i)

What is the risk of the portfolio when the


correlation between securities is +1.0?
a)
b)
c)
d)

ii)

i)

What is the expected return on shares?


a)
b)
c)
d)

ii)

What is the expected return on bonds?


a)
b)
c)
d)

iii)

9%
10%
1.9%
3.0%

29.44%
28.34%
25.44%
17.95%

What is the standard deviation of bonds?


a)
b)
c)
d)

Workbook

17.96%
14.56%
9.7%
15.71%

What is the risk of the portfolio when the


correlation between securities is -1?
a)
b)
c)
d)

12%
18%
15%
6%

427. Suppose you invest in 4 securities.


Company A has an expected return of 20%,
Company B 10%, Company C 12% and
Company D 9%. You have invested Rs.
40,000.
i. What more information is needed to find
out the possible return on the portfolio?

What is the standard deviation of stocks?


a)
b)
c)
d)

iv)

20%
25%
15.8%
13.2%

12%
18%
15%
6%

a)
b)
c)
d)

market value of the investment


beta of the shares
proportion of investment
none of the above

ii. Assuming that the portfolio is equally


weighted, what is the return on the
portfolio?
a) 12%
b) 10%
c) 12.75%
d) 11.5%

Investment Planning

49

428. A share pays nil dividends and its current


market price is 100. The possible selling
prices at the end of a year and the
probabilities are:

iii.

If the current market price of the stock is


37, what would you decide?
a) Buy and hold the share
b) Advise your friend to sell the share in
the market

430. Suppose a company sold an issue of bonds


with a 10 year maturity, a 1000 par value, a
10% coupon rate and semi annual interest
payments.

i.

What is the expected rate of return at the


end of the year?
a)
b)
c)
d)

ii.

8%
12%
10%
11%

What is the standard deviation of expected


returns?
a)
b)
c)
d)

Present value of the expected dividend in


the next 3 years.
a)
b)
c)
d)

ii.

At the end of the holding period, you expect


that the share price would be 34.73. What
is the present value of this expected price?
Your opportunity cost remains 12%.
a)
b)
c)
d)

50

6
4
5.28
4.1

24.72
31
32.
29

2 years after the bonds were issued, the


YTM of the bond is 6%. At what price
(approximately) the bond would sell in the
market?
a)
b)
c)
d)

ii.

10.55%
10.6%
10.9%
10.95%

429. Your friend offers to sell equity shares of a


company that paid a dividend of Rs. 2. You
expect the dividend to grow at the rate of
5% per year for the next 3 years. If your
holding period is 3 years and opportunity
cost is 12%, find out (assume the present
dividend is not received).
i.

i.

Rs.1100
Rs.1200
Rs.1232
Rs.1300

Suppose, after 2 years of issue, the YTM


rises to 12%, what would be the
approximate market price of the bond?
a)
b)
c)
d)

Rs.900
Rs.850
Rs.953
Rs.800

431. A mining company's reserves are fast depleting


and its sales are declining in recent years. Its
cost of mining is on the increase. Because of
these reasons, the company's earnings are
declining and dividends are expected to fall
constantly @ 5% per annum. Current dividend
is 5 and the discount rate is 15%.
i.

What is the value of the share?


a)
b)
c)
d)

ii.

Rs.20
Rs.21.25
Rs.23.75
Rs.24.25

As an financial advisor, under what


circumstances would you recommend
buying of the share?
a) If the company is expected to distribute
its large reserves in the near future
b) If the company is coming out with a fresh
issue to finance its expansion
c) If the company is taking efforts to cut
down its expenses
d) None of the above

Investment Planning

Workbook

432. You buy a share of ABC Ltd. for 21.40. You


expect it to pay dividends of 1.07, 1.1449
and 1.2250 in years 1, 2 and 3 respectively.
You also expect to sell it for the price of
26.22 at the end of 3 years. From this
information,
i.

ii.

435. Suppose you are asked to analyse two


portfolios having following characteristics.

10%
12%
11%
9%

Risk free rate is 7%. Return on market


portfolio is 15%. Standard deviation of the
market is 6%.
i)

Beta of A=1.5, Beta of B =1


Yield on 1 year T-bill is 8%. If market portfolio
is expected to yield 13% during the year, what
is the expected return on stock A?
a)
b)
c)
d)

12%
14%
15.5%
17%

ii)

12%
13%
16.5%
17%

Workbook

.2%
.3%
.4%
.-5%

1.33%
1.83%
1.38%
1.53%

What is the Treynor index on portfolio A?


a)
b)
c)
d)

4.5
2.5
6
6.5

436. Stock exchanges in India follow rolling


settlement system.

If the actual return on Stock A is 15%, what


was the residual error?
a)
b)
c)
d)

What is the Sharpe index for the market


portfolio?
a)
b)
c)
d)

What is the expected return on stock B?


a)
b)
c)
d)

iii)

Which is a better alternative?


a) Fund A
b) Fund B

2%
5%
3%
4%

433. You are given the following estimates of


share A and B.

ii)

0.433
0.533
0.563
0.536

What is the expected return on the share,


at the end of the holding period?
a)
b)
c)
d)

i)

What is the Sharpe performance measure


for fund B?
a)
b)
c)
d)

5%
6%
8%
7%

What is the dividend yield?


a)
b)
c)
d)

iii.

i.

Calculate the growth rate in dividends


a)
b)
c)
d)

ii.

434. You are trying to decide between two funds.


The risk free rate is 8%. Average return on
fund A is 18% and fund B is 16%. The
standard deviation is 20% and 15%
respectively.

i)

Under the rolling settlement system, any


transaction made on a particular day
necessarily results in delivery after a fixed
number of days.
a) True
b) False

Investment Planning

51

ii)

Currently stock exchanges in India follows,


T+5 settlements.

ii)

a) True
b) False
iii)

a)
b)
c)
d)

Switch cover from account period settlement


system to rolling settlement was facilitated
by which of the following?
a)
b)
c)
d)

Dematerialisation
Electronic fund transfer
Both (a) and (b)
None of the above

437. Radha owns the following portfolio.

ii)

ii)

8
10
12.8
15

0.1775
0.820
0.5795
1.01

iii)

0.70
1.70
0.72
1.27

What would be the market price (projected/


prospective) be if the P/E of the companies
in the same industry is 20?
a)
b)
c)
d)

What is the portfolio Beta?


a)
b)
c)
d)

What would be projected earnings per share


be?
a)
b)
c)
d)

What is expected return on the portfolio?


a)
b)
c)
d)

index gets cheaper


index rises
index remains steady
futures positions required payment of
MTM margin.

439. Your analyst, after a visit to Excel Cloth


Company has just announced its sales
forecast for the year 2004-14,40,000. If its
net income margin is likely to be 5% and
there are 100,000 equity shares outstanding.
i)

i)

Assuming that Ram adopts a long position in


index futures and the market prices rise as
expected, which of the following happens?

14
34
14.4
25.4

Assume that current market price is 10 and


the expected dividend payment is 0.10 per
share, what is the holding period return?

438. Ram is expected to receive Rs. 50 lakh by


gift from his mother-in-law. The money will
be received in a month's time. He plans to
invest 50% of his gift in shares. Recent trends
in the share prices indicate that the share
prices may go up. Impending elections may
dampen the spirit of investors and also if the
new government decides to adopts a stringent
economic policy. Based on the information
given above what should Ram do,

440. Subash has bought a 60 call option at 4 and


simultaneously sold a 70 call at 2 (lot size =
100). From this information, find out the
following:

i)

i)

If he wants benefit from the short time rally


in stock prices (when the money is not in
his hands)?
a) Buy stocks from spot market by
borrowing money
b) Buy long index futures/option
c) Short sell index futures
d) None of the above

52

a)
b)
c)
d)

54%
45%
33.33%
25%

What is the breakeven price?


a)
b)
c)
d)

Investment Planning

Rs.64
Rs.66
Rs.62
Rs.60

Workbook

ii)

What is the maximum profit expected from


this spread?
a)
b)
c)
d)

iii)

Rs.300
Rs.800
Rs.200
Rs.1100

What is the maximum loss expected?


a)
b)
c)
d)

Rs.400
Rs.300
Rs.200
Rs.100

iii)

a)
b)
c)
d)

a) You will be getting the Net Asset Value


of the scheme on a daily basis, so that
you can invest and disinvest everyday.
b) You will be in a position to trade on the
units intra-day as NAV is available on
real time basis.
c) You will be in a position to sell units of
what had been bought yesterday, i.e., a
buy today and sell tomorrow trading
system
d) None of the above.

A share broker applies for registration to


SEBI through a stock exchange of which
he is admitted as a member.
a) True
b) False

ii)

Sub-brokers are not responsible of


redressal of grievances of the investors as
it is the responsibility of brokers.
a) True
b) False

iii)

No stockbrokers is allowed to become the


member of more than one stock exchanges.

ii)

iii)

a) by physical delivery
b) in cash
c) gains/losses are settled in cash and the
contract value by physical delivery
d) none of the above
The option contracts have a trading cycle
of _________

Workbook

4 months
6 months
2 months
3 months

Since these funds are traded on the


exchange, the issuer is generally not
involved in purchase/resale of units.

444. In the NAV calculation of the mutual funds,


i)

The options are settled _________

a)
b)
c)
d)

Hedging
Arbitrage
Both (a) and (b)
None of the above

a) True
b) False

442. Answer the following questions with regard


to Index option trading on the stock
exchanges in India:

ii)

The advantage of investing in ETF is _____.


a)
b)
c)
d)

a) True
b) False

i)

American style options


European style options
Yankee options
None of the above

443. Your financial analyst recommends you to


make investment in Exchange Traded
Funds. With reference to this, you are asked
the following:

441. As per SEBI (Stock brokers and SubBrokers) Rules, 1992


i)

The index options traded are __________.

Which of the following factors are not


considered in valuation of unlisted share?
a)
b)
c)
d)

ii).

Net worth per share


Capitalisation rate
liquidity
Dividend payments

In case the latest balance sheet of the


company is not available, the unlisted
shares are valued _____________.
a)
b)
c)
d)

Investment Planning

at
at
at
at

50% of its fair value


25% of its fair value
0% of its fair value
90% of its fair value

53

iii)

Convertible debentures are valued on the


basis of:
a)
b)
c)
d)

Yield to maturity
Market value
Cost basis
Convertible portion like equity instrument
and non-convertible portion as debt
instrument

iv)

a) True
b) False
v)

445. Mutual funds offer various services to its


investors. They offer withdrawal,
reinvestment, switchover facilities etc.,
i)

The facility allowing investor to transfer on


a periodic basis a specified amount from
one scheme to another within a same fund
family is called __________.
a)
b)
c)
d)

ii)

447. Following statements are with regard to


borrowing by mutual funds.
i)

ii)

Banks
Non-banking financial institutions
Mutual funds
All of the above

iii)

a) On the happening of any event which,


in the opinion of the trustees, requires
the scheme to be wound up
b) If 75% of the unit holders of a scheme
pass a resolution that the scheme to be
wound up
c) If the board so directs in the interest of
the unit-holders.
d) Any of the above

Mutual funds are allowed to trade in


derivatives for the purpose of hedging,
portfolio balancing and speculation.

a) True
b) False

54

ii)

Mutual funds are allowed to invest in overseas


securities, subject to the approval of Board,
with out any investment cap or ceiling.

Which of the following statement is not


correct?

448. i) A scheme of mutual fund may be wound


up, after repaying the amount due to the
unitholders ___________.

Mutual funds are allowed to participate in


securities lending.

a) True
b) False

Not exceeding one year


Not exceeding three months
Not exceeding six months
For any period

a) SEBI restricts the quantum of borrowings


by mutual funds
b) Temporary liquidity needs can be met
through borrowings
c) Corpus can be increased through
borrowing
d) Time limit is specified for the borrowing
and repayment.

a) True
b) False
ii)

Mutual funds can borrow money for a period


_________
a)
b)
c)
d)

446. Mutual funds are investment pools offering


benefits of diversification especially to small
investors. Which of the following statements
about mutual fund investments are correct?
i)

A mutual fund scheme can invest in another


scheme of same asset management
company.
a) True
b) False

Systematic investment Plans


Systematic withdrawal plans
Automatic reinvestment plans
Systematic transfer plans

Investors can avail loan upto certain


percentage of value of the holding in mutual
funds units. Loans against the units can not
be availed from __________
a)
b)
c)
d)

With the specific approval from SEBI,


mutual funds invest in unlisted security of
an associate or group company of sponsor.

Mutual funds can guarantee return on any


of its scheme if ____________
a) the asset management company fully
guarantees the return.
b) the sponsor fully guarantees the return
c) both (a) & (b) are true
d) Mutual fund can not guarantee any return.

Investment Planning

Workbook

449. Following questions pertain to investment


in Public Provident Fund, which is a popular
savings cum tax saving instrument.
i)

Which of the following statements is not


correct?
a) Non-resident Indians can not open an
account
b) Continuance of an existing PPF account,
opened at a time when the account
holder was resident, will be allowed.
c) Subscription to an existing PPF account,
opened at a time when the account
holder was resident, will be allowed on
repatriation basis
d) Hindu Undivided Families can open PPF
account.

ii)

Read the following statements carefully and


indicate the correct ones.
a) In case of join nominations, it is not
possible to allocate the percentage of
benefits to each nominee.
b) The PPF account can be extended only
when the account holder continues the
contribution
c) Balance in PPF account is not subject
to attachment by Income Tax authorities
d) None of the above.

451. i) If you are planning an investment in a


remote, rural area where you have a hunch
that the national high way would come up
in the vicinity and the place would be a
township preferred by Business Process
Outsourcing (BPO) entrepreneurs. What is
this category of real estate investment?
a)
b)
c)
d)
ii)

iii)

ii)

Non Resident Indians are allowed to invest


in the bonds
a) True
b) False

iii)

There is lock-in period


a) true
b) false

Workbook

As per section 54 of The Income Tax Act,


1956 no tax liability on excess of capital
gains on sale of old house over the cost of
financing new house which is purchased/
constructed as per the conditions stipulated.
a) True
b) False

iv)

Which of the following arrangements


provide the perpetual right to occupation or
right to use for a set time or period each
year for certain term.
a)
b)
c)
d)

Premature withdrawal is not allowed.


a) True
b) False

Indexation benefits are not considered while


computing the capital gains realized on sale
of land or house property held for not more
than 36 months.
a) True
b) False

450. Following questions relate to RBI savings


bonds.
i)

passive secure investment


development investment
speculative investment
both (b) and (c)

Lease
Mortgage
Time share
Lien

452. As fund manager in a large financial


institution, Mr. X invests money in short-term
money market instrument, fixed and floating
interest securities, equity and preference
shares and bank and corporate deposits.
i)

Which of the following instruments are


useful in managing interest rate risks?
a)
b)
c)
d)

Investment Planning

Forward Rate Agreements


Interest Rate Swaps
Commercial Paper
Both (a) & (b)

55

ii)

Repurchase agreements are essentially


a)
b)
c)
d)

Secured loans
Unsecured loans
Long term investment
Carries no interest

453. Assume, today is 1st February, 2004, you


have just concluded a business deal and
received an advance of 10 lakhs. You would
require the money for the commencement
of operations and other payments by 1st
April 2004. You are told that the funds can
be parked in commercial paper (CP) which
would give a better return compared to
deposit in banks.
i)

ii)

a)
b)
c)
d)
iii)

Which of the following is not the feature of


the CP?
a)
b)
c)
d)

iii)

i)

ii)

What are the economic factors?


a)
b)
c)
d)
e)

ZCYC depicts the relationship between


____________.
a) interest rate and maturity for a asset of
bearing bonds
b) interest rate and maturity for a set nonconvertible bonds
c) interest rate and inflation for a set of noninterest bearing bonds
d) inflation rate and maturity for a set of
interest bearing bonds
e) None of the above.

Issuing companies
SEBI
RBI
Banks

454. Interest rates in an economy are influenced


by various factors. These factors are broadly
classified in to economic factors and noneconomic factors.
i)

X sells an interest-bearing instrument to Y


on 15.3.2003. He received interest from the
security on 1.1.2003. The consideration of
sale received by X would _________.

455. Zero coupon yield curve (ZCYC) is a widely


used measure for bond valuation.

CPs are credit rated


CPs are unsecured instruments
CPs are liquid instruments
CPs are not bought back.

The norms for CP issue is prescribed by____.


a)
b)
c)
d)

par
discount
premium
none of the above

a) exclude the accrued interest from the


date of last interest payment till date of
sale
b) be inclusive of accrued interest till the
date of sale
c) as the security is sold, Y will be receiving
the future interest and no portion of it
will be shared with X
d) interest amount will be shared equally
between X and Y.

CPs have ___________.


a) no maturity
b) short maturity but generally rolled over
for a fairly long period
c) fixed maturity
d) fixed maturity but for more than a year.

ii)

Due to changes in interest rates, there may


be a situation where there is a price gap
between new and existing securities traded
in the market place. When the yield exceeds
the return from the coupon payment alone,
the security is said to be traded at ______.

ZCYC is ____________.
a) useful for estimating the credit risk
b) useful for estimating the premium to be
charged for default risk
c) used as benchmark yield for risk free
securities
d) both (b) & (c)

Inflation Rate
Current Account Deficit
Currency movements
Only (a) & (b) and not (c)
(a), (b) and (c)
iii)

ZCYC is generally positively sloped.


a) True
b) False

56

Investment Planning

Workbook

456. The market place where initial issue of Fixed


Income Securities, like equities, is known
as primary market. The secondary market
deals with securities already issued.
i)

In the primary market, the securities are


issued through _________.
a)
b)
c)
d)

ii)

Rs.
Rs.
Rs,
Rs.

1,00,000
1000
10,000
100

Subsidiary General Ledger (SGL) is ______


a) Holding of investment in government
securities in electronic form.
b) A facility provided by the RBI.
c) Necessary for Financial Institutions,
Banks, Intermediaries like Primary
dealers and NBFCs to deal in government
securities.
d) All of the above

iv)

Suppose you have invested in 1000 shares


of an infotech company and you are afraid
that the prices would decline. What would
be your strategy to protect the value of your
portfolio?
a)
b)
c)
d)

ii)

Sell a put option


Buy a put option
Buy futures
None of the above

Do you think that the risk on the portfolio be


hedged with call option? In that case, what
would you do?
a)
b)
c)
d)

Workbook

Holding only call or put options


Having the securities and no options
Holding securities and writing the call
Holding securities and writing the put

Naked position in the derivatives market


refers to _________
a) No derivative instruments in portfolio
b) Having the securities and equivalent
derivative instruments
c) Holding securities and buying the call
d) Only call or put or futures open position

458. Short selling means selling without


securities. This is riskiers and is the reason
why mutual funds and financial institutions
are not allowed t o engage in these
transactions. Suppose you, an individual,
expect that the market would decline, are
tempted to sell some overvalued shares at
the existing level without owning them.
i)

Under which of the following circumstances,


you would be in a position to benefit from
the transactions?
a) When the index gains on the next day
and the exchange follows T+1 settlement
system
b) When the index declines after 3 days and
the exchange follows T+1 settlement
system
c) When the index declines after a week
and the exchange follows T+3 settlement
system
d) None of the above

457. Hedging is a way of reducing the risks


involved in holding an investment. There are
different instruments available for hedging
and risk management.
i)

Protective option (call or put) means ______


a)
b)
c)
d)

The minimum subscription amount in the


primary market in case of government
securities is ___________.
a)
b)
c)
d)

iii)

Public issue
Private placement
Tender/Auction
Any of the above

iii)

ii)

Your financial advisor tells that trading in


derivatives is less riskier than selling short.
What would be your strategy to benefit from
your expectations?
a)
b)
c)
d)
e)

Write a call
Write a put
Long call
Long put
Either a or d

Buy call option


Sell Call option
Hedging through call option not possible
Hedging requires a combination call and
put options.
Investment Planning

57

iii)

For hedging large portfolios in the event of


market decline, which of the following would
be suitable?
a)
b)
c)
d)
e)

Derivatives on stocks
Derivatives on Index
Exchange Traded Funds
All except (a)
All except (c)

459. Mutual funds collect money from investors.


It is important that the interests of small
investors in mutual fund schemes are
protected. Mutual fund structure takes into
consideration this aspect.
i)

a)
b)
c)
d)
iii)

Maximum 40%
Minimum 40%
Maximum 75%
Minimum 25%

Suppose you subscribe to a close ended


fund at the time when the units are offered
to public, what is the disinvestment avenue?

58

Stock Exchanges
Repurchase facility offered by the fund
(a) and (or) (b) above
None of the above

max (0, X-S)


max (0, S-X)
Min(0, X-S)
Min(0, S-X)

Intrinsic value of the put option is


max (0, X-S)
max (0, S-X)
min (0, X-S)
min (0, S-X)

At expiry, options have


a)
b)
c)
d)

Only Time value


No Intrinsic value
Both time and intrinsic value
No time value

462. Which of the statements given below is not


correct with regard to futures transactions.
i)

460. As an investor there are different schemes


of mutual funds from which you can choose
and park your investible funds. It is desirable
to read the offer document to know the
salient features of the schemes before
committing the funds.

a)
b)
c)
d)

Intrinsic value of the call option is _______


a)
b)
c)
d)

iii)

Asset Management Company


SEBI
Board of Trustees
Sponsors

What is the contribution of sponsor to the


net worth of asset management company?
a)
b)
c)
d)

i)

i)

a)
b)
c)
d)

The custodian is appointed by

Tax savings scheme


Real Estate Funds
Equity Funds
Index Funds

461. As an investor, it is necessary to know the


nuance of option valuation. It will be useful
to trade, hedge or arbitrage in options.

ii)

Companies Act, 1956


SEBI Act, 1992
Indian Trust Act, 1882
All of the above

The concept of tracking error is applied to


measure performance in case of _______
a)
b)
c)
d)

Which of the following legislations are


applicable to Mutual funds in India?
a)
b)
c)
d)

ii)

ii)

Benefit of dealing in futures contract is


_______.
a)
b)
c)
d)

ii)

leveraging exposure
cash flow management
portfolio substitution
benefit from premium payments

Transaction in futures market involve


________.
a) Payment of initial margin
b) Payment of mark to market margin
c) Payment of premium

iii)

Which of the following conditional orders


can be placed with the brokers/trading
members as per their requirements?
a)
b)
c)
d)

Investment Planning

orders based on time conditions


orders based on price conditions
both (a) and (b)
None of the above

Workbook

463. Tax is an important consideration of any


investor. Following are some questions based
on the taxation aspects of investment.
i)

iii)

iii)

i)

Securities market in India are also regulated


by ____________.
a)
b)
c)
d)

Workbook

Department of Company Affairs


Securities Appellate Tribunal
Department of Economic affairs
All of the above

When new shares are offered at an issue


price which may be payable by progressive
amounts is known as __________.
a)
b)
c)
d)

ii)

464. SEBI is the regulator of capital markets in


India and has statutory powers for protecting
the investors and promoting the
development of securities market. The
following questions address the aspects of
regulations in the Indian scenario.
i)

Issue, allotment and transfer of securities


Standard of disclosure in public
Underwriting of issues
All of the above

465. Proprietary and Public companies are the


two commonly found type of the companies
in any country. Former manages with the
resources available with them the latter has
outside funding in its balance sheet.

The cost of rights entitlement in the hands


of the original holder is __________.
a) equal to the offer price of the rights share
b) nil
c) will be taxed as long term capital gains
if the sale price exceeds the cost price.
d) None of the above

Companies Act lays down rules and


regulations regarding ____________.
a)
b)
c)
d)

Which of the following statements is correct?


a) Cost of acquisition of bonus shares is
nil
b) Cost of acquisition of bonus shares is
equal to the market value of share on
the date of allotment.
c) Expenditure incurred on transfer of
shares is excluded in full value
consideration computation
d) None of the above

Derivatives on gold related securities and


government securities that are traded on the
stock exchanges are regulated by _______.
a) Reserve Bank of India in consistent with
the guidelines issued by SEBI
b) SEBI in consistent with the guidelines
issued by SEBI
c) Companies Act
d) Department of Company Affairs

Cost of acquisition is _____________.


a) Value spent for acquiring the investment
b) Value spent on acquiring the investment
plus the capital expenses incurred for
acquisition
c) Value spent on acquiring the investment
minus the capital expenses incurred for
acquisition
d) Value spent for acquiring the investment
plus depreciation

ii)

ii)

Application money
Allotment money
Calls
None of the above

Share holders in a limited company are not


liable for _____________.
a) The paid amount of capital in that
company
b) The assets of the limited company
c) The debt of the company if his shares
are fully paid-up.
d) The goodwill of the company

iii)

The companies in which significant amount


of shares are held by the parent which also
exerts some influence in the activities is
known as _____________.
a)
b)
c)
d)

Investment Planning

Wholly owned subsidiaries


Partly owned subsidiaries
Associated companies
Unlimited companies

59

466. Covered call writing is a strategy by riskaverse investors. Consider an investor who
writes a covered call on XYZ share. Spot
price is 38, exercise price is 40 and 3 month
writing call on XYZ share is traded at 3.
i)

What is the maximum profit realizable from


this strategy?
a)
b)
c)
d)

iii)

2
4
5
0

a)
b)
c)
d)

i)

60

ii)

A speculator is a trader who _________.


a)
b)
c)
d)

5.3%
14.3%
8.57%
2.63%

The break even point of the strangle is :


a) when the stock price is between 88 and
76
b) when the stock price is between 73 and
92
c) when the stock price is either 73 or 92
d) none of the above

Hedger is a trader
a) who enters derivatives market in order
to reduce a pre existing risk.
b) Hedger is a trader who enters derivatives
market in anticipation of a need in the
near future.
c) Whose net wealth change, at the time the
derivative contract expires, is expected to
be zero. (Perfect hedge assumed).
d) All of the above

5.3%
14.3%
8.57%
2.86%

467. Strangle consists of a put and a call with


the same expiration date and the same
underlying. An investor has a call with an
exercise price of 85 and a put with an
exercise price of 80. The call price is 3 and
put price is 4.

the straddle buyer will exercise the call


the straddle buyer will exercise the put
the straddle seller will deliver the stock
none of the above

468. In the derivatives market, hedgers,


speculators and arbitrageurs trade.
i)

What is the unannualised rate of return on


the investment in shares alone (assume that
no calls is written)?

5
3
7
0

If the stock price is 77 :


a)
b)
c)
d)

What is the unannualised rate of return if


the share price rises to 42?
a)
b)
c)
d)

iv)

iii)

38
35
41
37

If you have taken short position in the above


strangle, what would be the profit it the
share price is 83?
a)
b)
c)
d)

What is the initial net cash flow incurred at


the time of investment?
a)
b)
c)
d)

ii)

ii)

assumes no risk for profit


assumes risk for profit
does not incur losses at all
has a long position in cash market and
long position in derivatives market.

469. In the primary market today, the number of


companies coming out with issues take the
book building route.
i)

Green shoe option :


a) allows the underwriter to retain a portion
of the oversubscription in the book
building route
b) is refund of money when the issue in
undersubscribed
c) acts as a balancing factor and stabilize
the prices
d) both a & c

Investment Planning

Workbook

ii)

The issuer gets the flexibility to change the


pricing as per market conditions when the
book building is done as per floor price route

ii)

a)
b)
c)
d)

a) True
b) False
iii)

A retail investor under book building process


is one _____________
a) who applies for 1000 shares to applications
for shares worth Rs. 10000 or less
b) who applies for 1000 shares to applications
for shares worth Rs. 50000 or less
c) who applies for 1000 shares to application
for shares worth Rs. 25000 or less
d) None of the above

470. At the beginning of the year Ray decided to


take Rs. 50,000 in savings out of the bank
and invest it in a portfolio of stocks and
bonds; Rs. 20,000 was placed into common
stocks and Rs. 30,000 into corporate bonds.
A year later, Ray's stock and bond holdings
were worth Rs. 25,000 and Rs. 23,000
respectively. During the year Rs. 1,000 cash
dividends was received on the stocks and
Rs. 3,000 in coupon payments was received
on the bonds. The stock and bond income
was not reinvested in Ray's portfolio.
i) Find out the return on Ray's stock
portfolio during the year.
ii) What was the return on Ray's bond
portfolio during the year.
iii) Find out the return on Ray's total portfolio
during the year.

iii)

iv)

a)
b)
c)
d)

Workbook

20%
17.21%
2%
13.51%

28.57%
33.33%
40%
20%

What is the return on the portfolio?


a)
b)
c)
d)

21.19%
17.48%
19.16%
10.96%

472. Below is the data on two companies in the


same industry.

i)

Dividend yield of Acharya is ____.


a)
b)
c)
d)

ii)

0.3
0.55
0.12
0.22

P/E Ratio of Bhushan is _______.


a)
b)
c)
d)

iii)

What is the weight of Security B?

10.71%
12%
11.32%
13%

Determine the return on Security A.


a)
b)
c)
d)

471. Determine the expected return on the


following portfolio:

i)

The expected return on Security C is ____.

600
343
182
273

Which one of the above is the better


investment propostions?

473. Shivatsav & Co. pays a dividend of Rs. 1.50.


The current market price is rs. 25. Under
market estimation with a growth factor of 11%
and a discount rate of 15%; and in reality, with
a growth factor of 7% and a discount rate of
15% how is the share rated, over-priced or
under-priced, to be sold or bought?
a) Sell
b) Buy

Investment Planning

61

474. Calculate the yield value of a treasury bill


with a face value of Rs. 1,50,000. It has 125
days to maturity and the market value today
is Rs. 1,44,231.
a)
b)
c)
d)

9.85%
12.1%
11.7%
10.5%

475. The information on growth schemes of two


mutual funds are given below:

Under assumption of risk free rate at 9%,


interpret the results of Jensen, Treynor and
Sharpe index.
476. Ashwin Yadav proposes to purchase a
property for giving it on rent. (Ignore taxation
and gearing). He expects to receive Rs. 55,000
in net receipts each year for six years and to
sell the property for Rs. 8,50,000 at the end of
the six year period. If the expected return is
15% what would be the value of the property?
a)
b)
c)
d)

i)

What is the rate of return on portfolio A


and B?

ii)

Calculate the CoVariance.

iii)

Calculate Standard Deviation of the


Portfolio for equal investment.

480. The effective interest rate earned per rupee


______ as the periods of compounding
increase
a)
b)
c)
d)

Increases
Decreases
Remains same
Decreases for some time and then
increases
e) Data insufficient
481. The term Efficient FrontierIn is contained
in___________.
a) Technical Analysis
b) Modern Portfolio Theory
c) Value Investing Theory

Rs.8,00,000
Rs.7,56,700
Rs.5,75,625
Rs.4,48,678

477. For an asking price of a property at Rs. 9,50,000


with an estimated net income of Rs. 1,25,000
at a market yield of 12%, calculate the value of
the property on capitalization approach.
a)
b)
c)
d)

479. Calculate for securities A and B based on


the data given below:

Rs.10,42,000
Rs.12,56,000
Rs.9,88,888
Rs.8,98,850

482. In India, Mutual Funds have recently moved


to the concept of ______ AUM calculation.
a) Monthly average
b) Month end
c) Fortnightly average
483. An Asset Management Company must have
a minimum corpus of Rs. _______crores.

478. With the following data shown in the table


below, compute the risk on the portfolio.

a)
b)
c)
d)

5
15
25
10

484. __________ funds pay a Dividend


Distribution tax on dividends.
a) Equity
b) Index
c) Debt

62

Investment Planning

Workbook

485. A perfectly diversified portfolio will fully


eliminate ___________ risk.

492. Data on two stocks is given for 2 different


scenarios:

a) Systematic
b) Unsystematic
486. Classifying an investment as a long term
investment depends primarily on;
a) the length of time the investor expects
to hold the investment
b) the amount of the investment
c) whether a liquid market exists for selling
the investment.
487. In India Futures contracts in ______ may
be settled by delivery.

Find Beta of both stocks


a)
b)
c)
d)

1.8, 0.60
25, 0.75
1.5, 0.25
2.2, 0.5

493. Data on a mutual fund is given:

a) Commodities
b) Stocks
c) Stock Index
488. Mr. A deposits Rs. 10,000 in his own PPF
account and same amount in his wife's
account. How much maximum amount can
be deposit in his newphew's name?
a)
b)
c)
d)

Rs. 20,000
Nil
Rs. 70,000
Rs. 60,000

489. A Rs. 100 par value bond having 10%


coupon rate will mature after 7 years. Find
the value of the bond if the discount is 8%.
a)
b)
c)
d)

a)
b)
c)
d)

1.05, 0.10, 4.25


1.33, 0.04, -4.25
1.10, 0.15, -3.75
1.46, 0.09, 3.75

494. Secure Funds has 3 investments in its


portfolio. Details as given below. Calculate
the expected return of the portfolio.

109.85
110.41
108.75
110.60

490. A 5 year annual annuity has a yield of 6%.


What is the duration?
a)
b)
c)
d)

The risk free rate is 9%; Calculate Treynor,


Sharpe and Jensen measures.

2.88 years
2.55 years
3.16 years
1.35 years

495. From the following information, calculate the


expected rate of return of a portfolio.

491. The price of Steller Ltd. is currently Rs. 40.


The dividend next year is expected to be
Rs. 4.00. Required return on the stock is
12%. Find the expected growth rate under
the Constant Growth model.
a)
b)
c)
d)

Workbook

2.00%
2.25%
1.90%
2.75%

Investment Planning

63

iii. What would be the average return on the


portfolio during the period?

496. The following information is given


Risk-free rate of return 8%
Expected rate of return on market portfolio
16% Beta of a security 0.7
i) Find out the expected rate of return of
the security
ii) If another security has an expected
return of 20% what must be its beta?
497. Calculate the expected rate of return of a
portfolio from the given information.
Risk-free rate of interest 8%.
Expected return of market portfolio 18%.
Standard deviation of an asset 2.8%.
Market standard deviation 2.3%.
Correlation co-efficient of portfolio with
market 0.8%.

500. V V Ltd. has an expected return of 22% and


Standard Deviation of 40%. XY Ltd. has an
expected return of 24% and Standard
Deviation of 38%. VV has a beta of 0.86
and XY has a beta of 1.24. The correlation
coefficient between the return of VV and XY
is 0.72. The Standard Deviation of market
return is 20%.
i. Is investing in XY better than investing
in VV?
ii. If you invest 30% in XY and 70% in VV,
what is your expected rate of return and
portfolio Standard Deviation?
iii. What is the market portfolios expected
rate of return and how much is the riskfree rate?
iv. What is the beta of portfolio if VV's weight
is 70% and XY's weight is 30%?

498.

i) Which securities would you select?


ii) Assuming perfect correlation, analyse
whether it is preferable to invest 75% in
security A and 25% in security C.
499. Stocks A and B have the following historical
returns:

With 50% held in Stock A and 50% in Stock


B. Calculate the following.
i. The average rate of return for each stock
during the period 2000 through 2004.
ii. What would be the realized rate of return
on he portfolio in each year from 2000
through 2004?

64

Investment Planning

Workbook

Answers

Workbook

Investment Planning

65

Answers

66

Investment Planning

Workbook

Answers

Workbook

Investment Planning

67

Answers

68

Investment Planning

Workbook

Answers

Workbook

Investment Planning

69

Explanatory Answers
1. Ans, [c].

35.

Ans, [c].

2. Ans, [d].

36.

Ans, [d].

3. Ans, [a].

37.

Ans, [e].

4. Ans, [d].

38.

Ans, [d].

5. Ans, [d].

39.

Ans, [b].

6. Ans, [a].

40.

Return on portfolio is sum of (expected return of


individual asset x probability)
Ans, [b].

41.

Ans, [e].

42.

Price = Dividend / ( rateof return - rate of growth of


dividend) Next year dividend = 3.85 x (1.07)=4.1195
40 = 4.1195 / (r-.07) Solving for r gives 17.298
Ans, [d].

43.

Apply the same concept as above.


Ans, [c].

14. Ans, [d].

44.

Ans, [b].

15. Ans, [d].

45.

Ans, [a].

16. Ans, [a].

46.

Ans, [b].

17. Ans, [b].

47.

The actual cost is Premium paid ( Rs. 300) + Exercise


value (50 x 100)= Rs. 5300
While the actual market value on the date of exercise is
(57 x 100)= Rs 5700 , which leaves a profit of Rs. 400.
Ans, [b].

48.

Market value after 6 months for five hundred shares


of B is (35 x 500) = 17500
Exercise price after 6 months for five hundred shares
of B is (45 x 500) = 22500
Profit in put option is (22500 3000) = 19500,
Rs 3000 is the premium paid for put option.
Value in put option is 19500 17500 = Rs 2000.
Ans, [a].

26. Ans, [e].

49.

Ans, [c].

27. Ans, [e].

50.

The exercise price is not profitable, so he would choose


to go with market price which is higher. But he loses the
money paid in form of premium ie Rs 225.
Ans, [a].

51.

Because the market price is lower than exercise price


the buyer would prefer to buy in market than to
exercise the option. Which leaves the call writer with
the profit of premium collected ie Rs 625.
Ans, [a].

52.

Ans, [b].

53.

Ans, [d].

54.

Ans, [a].

55.

Ans, [c].

56.

Ans, [a].

7. Ans, [d].
8. Ans, [d].
9. Ans, [d].
10. Ans, [c].
11. Ans, [a].
12. Ans, [d].
13. Ans, [b].

18. Ans, [c].


19. Ans, [b].
20. Ans, [a].
21. Ans, [a].
22. Ans, [a].
23. Ans, [a].
24. Ans, [b].
25. Ans, [b].

28.

d/(d+e)=.4, solving for e taking d to be 2lacs gives


equity portion to be 3lacs. So the ROE is 10%.
Ans, [c].

29.

Book value of share is 2000/40 ie 50, Market price


is twice the book value so it is 100.
Ans, [b].

30.

Ans, [d].

31.

You need to have negative correlation to have


hedging.
Ans, [b].

32.

Ans, [a].

33.

Ans, [d].

34.

Ans, [d].

70

Investment Planning

Workbook

57.

Ans, [b].

96.

Ans, [e].

58.

Ans, [a].

97.

59.

Ans, [a].

60.

Ans, [a].

61.

Ans, [c].

62.

Ans, [a].

63.

Ans, [b].

The client will benefit if the market increases. Her


portfolio, because it is diversified, should be highly
correlated and move with the market. An index option
also moves with the market and, therefore, would
be a good hedge vehicle. A put should be used,
because it will increase in value if the market should
decline, thereby offsetting any losses on the portfolio.
Ans, [c].

64.

Ans, [c].

98.

65.

Ans, [e].

66.

Ans, [a].

Realized Return - Risk Free Return Sharp Index =


Standard Deviation of Portfolio (0.19 - 0.08). 0.47826
= 0.23.
Ans, [c].

67.

Ans, [c].

99.

68.

Ans, [c].

69.

Ans, [d].

Unless the correlation coefficient bet ween the stocks


is equal to one, the standard deviation for the
portfolio will be lower than the weighted average
standard deviation for the portfolio.
Ans, [d].

70.

Ans, [b].

71.

Ans, [a].

72.

Ans, [c].

73.

Ans, [a].

74.

Ans, [e].

75.

Correlation coeff: = covariance / ( sigma a x sigma b)


Ans, [e].

76.

Ans, [c].

77.

Ans, [a].

78.

Ans, [a].

79.

Ans, [b].

80.

Ans, [a].

81.

Ans, [b].

82.

Ans, [a].

83.

Ans, [b].

84.

Ans, [a].

85.

Ans, [b].

86.

Ans, [b].

87.

Ans, [a].

88.

Ans, [a].

89.

Ans, [b].

90.

Ans, [c].

91.

Ans, [e].

92.

Ans, [c].

93.

Ans, [c].

94.

Ans, [d].

95.

Ans, [d].

Workbook

100. The intrinsic value of a share of common stock is


equal to the discounted present value of its cash
flows.
Ans, [e].
101. The option is out of the money; therefore the intrinsic
value is zero. The premium of an option does not
affect the intrinsic value of the option.
Ans, [a].
102. Arbitrage Pricing Theory uses multiple regression
(many factors) to determine a model or formula that
has numerous factors. This model is then used to
determine the value of a security. The CAPM is
based on the single factor of Beta, which a portfolio.
Ans, [a].
103. A call option has unlimited price potential which
means that writing a call without the stock as a hedge
will provide the greatest loss potential.
Ans, [b].
104. The market risk premium is the additional return for
accepting the risk of the market. If the market
premium increases with all else remaining the same,
then the price of the stock would have to decrease.
An increase in the market premium would also
increase the discount rate used to value the stock.
This higher discount rate will cause the present value
of the cash flows to be smaller.
Ans, [d].
105. PV = Rs.1.36
FV = (Rs.2.00)
N= 5
i = 8.02
d1 1.082 (2.00) 2.16
Value of common stock = Rs.54.00
RRR - g 0.12 - 0.08 0.04
RRR = Required Rate of Return
g = growth rate
Ans, [e].

Investment Planning

71

106. Sharpe uses standard deviation and Treynor uses


beta.
Ans, [d].
107. PV = (Rs.1,147.20), PMT = 80, N =10, PV = Rs. 1050,
FV = Rs.1,000, i = 65.45
The price of a bond is inversely related to changes in
interest rates. Therefore, #1, #2, and #3 are all correct.
Ans, [a].
108. Non-diversifiable risks or systematic risks are those
that affect the entire market, including market risk,
interest rate risk, and purchasing power risk.
Ans, [a].
109. ADRs are used to trade foreign securities in the U.S.
ADRs are foreign shares denominated in U.S. dollars
and do not eliminate currency risk.
Ans, [d].
110. Statements 1, 2 and 3 are true. Statement 4 is false
because tax swaps generally take advantage of
capital losses by selling bonds, which have been
devalued by increasing interest rates.
Ans, [a].
111. Statement 1 is true. It is not necessary to have
negatively correlated assets; it is only necessary to
have assets that have a correlation less than positive
one (+1); thus, Statement 2 is false. Statement 3 is
false, because diversifying across asset types is
more, not less, effective than within an asset type.
Statement 4 is false, because all the input variables
in Statement 1 help to create the efficient frontier.
Ans, [c].
112. Convertible bonds generate current income from
coupon payments and allow for growth through the
stock conversion feature. Options [a] & [b] provide
income only and Option [c] is designed for growth.
Ans, [d].

117. Above the line would indicate a higher than expected


return for the given risk level. On the line would
indicate an expected return for the given risk level.
Below the line would indicate lower than expected
return for the given risk level.
Ans, [b].
118. A debenture is an unsecured corporate debt.
Ans, [e].
119. The goal of immunization is to match the investment
time horizon with the duration of the portfolio. Since
the duration of a coupon bond is less than its
maturity, only the 9-year bond might immunize the
portfolio. The other two choices will not immunize
the portfolio. The best choice would be a 7-year
zero-coupon bond; however, this choic e is not
available.
Ans, [b].
120. Systematic risk cannot be eliminated, thus Statement
2 is false. Beta only measures systematic risk;
Statement 5 is false. All other statements are true.
Ans, [b].
121. Rupee cost averaging is when an equal rupee
amount is inves ted periodic ally. This does not
prevent capital losses but can lower the average cost
per share due to periods of decline in the stock price.
Ans, [b].
122. CFo = (Rs.9,200) CF1 = Rs.600 CF2 = Rs.2,300
CF3 = Rs.2,200 CF4 = Rs.6,800 CF5 = Rs.9,500
IRR = 24.18%
Ans, [c].
123. Statement 4 is false, because if the cost of capital is
les s than the IRR, then the project should be
accepted (NPV > 0).
Ans, [d].

113. The exercise price for a put is the price at which you
can sell the stock. Thus, a price of Rs.55 will be
assured if she buys a Rs.55 put.
Ans, [c].

124. Total annual income = Rs.26,500 = 6.63% annual


yield FMV @ June 30 last year Rs.400,000 Change
in value Rs.85,000 FMV @ June 30 last year =
Rs.400,000 = 21.25% appreciation (Before tax)
Ans, [d].

114. V = d0 (1 + g) = Rs.2.30 (1.04) = Rs.47.84 (value


using dividend growth model), k - g 0.09 - 0.04 Since
the value of Rs.47.84 is greater than the current
market price, the stock is underpriced in the market.
Tom should purchase the stock.
Ans, [a].

125. The bond with the longest duration has the greatest
interest rate risk. The greater the duration, the more
effect a change in interest rates has on the bond's
value. A zero coupon bond's duration equals its time
to maturity.
Ans, [a].

115. One of the first steps that must be taken in planning


for wealth accumulation is to determine the risk
tolerance of the investor.
Ans, [d].

126. A zero-coupon bond's duration equals its maturity.


Coupon bonds (the remaining options ) have
durations less than their maturities. Higher duration
bonds are subject to more price volatility than lower
duration bonds.
Ans, [b].

116. Of all the choices, only unexpected corporate earning


growth would have a significant positive impact on
the value of a stock. Under the Efficient Market
Hypothesis, any expected changes would already
be reflected in the price of the common stock.
Unexpected increases in inflation would increase the
discount rate and reduce the value of the fund.
Ans, [c].

72

127. A longer-term bond will be subject to more inflation


risk. Since the quality of the bond is high, the level
of default risk is low.
Ans, [a].

Investment Planning

Workbook

128. Selling a call option will allow her to generate income


from the option premium with little risk since she does
not expect the stock to continue to increase. If the
stock does exceed Rs.50, she would be paid what
she wants for the stock.
Ans, [b].

136. Net Present Value (NPV) is defined as the difference


between the discounted inflows and the initial
outflow. For example, if the discounted cash inflows
were Rs.10 and the initial outflow was Rs.8, then
the NPV would be Rs.2.

129. The statement in the question is referring to the


definition of the semi strong form of the efficient
market hypothesis.
Ans, [c].
130. N = 35 (17.5 x 2)i = 4 (8/2)PMTOA = Rs.37.50
(Rs.75/2) FV = Rs.1,000, PV = (Rs.953.34)
Ans, [c].
131. Generally, investors evaluate performance of
inves tments bas ed on risk adjus ted returns .
Therefore, options [a] and [b] must be wrong since
they only address one aspect of the risk-return
relations hip. Treynor and Sharpe ratios are
performance measures in which the higher the ratio,
the better the risk adjusted return. No calculation is
needed for this question. Option [d] is the only
reasonable answer. It is the only one that indicates
that a fund s hould be c hosen, bec aus e the
performance measure is high (not low).
Ans, [d].
132. To immunize a bond portfolio, an investor should
match the duration of the bond portfolio to the
investment time horizon.
Ans, [b].
133. Mortgage-backed securities are subject to the same
risk as bonds plus the risk of prepayment.
Ans, [d].
134. Question 1 should always be asked. Question 2 is a
reasonable question, since it provides the planner with
a concept of the client's investment experience
leading to suitability. This type of analysis is a part of
data gathering. Question 3 goes to suitability and
should be asked, or the age should be determined
relative to risk and time horizon. The question relates
to the first two stages of data collection and goals.
Reinvestment of dividends does not go to suitability
and will be addressed after the investment decision
is made. Below is the CFP Board of Examiners'
clarification of the question: (1), (2), and (3) all relate
to the second step of the financial planning process,
namely, "Gathering client data and determining goals
and expectations". The first step is "Establishing the
client-planner relationship".
Ans, [c].
135. Duration is a present value, time weighted measure
of payback. All of the factors listed are important
components in determining duration.
Ans, [e].

Ans, [d].
137. This question is regarding the constant dividend
growth model for determining the value of a stock.
The following formula is used for the constant
dividend growth model:
D1P0 = k - g where:
P0 = Price for the security. D1 = The dividend paid
at period 1.
k = The investor's required rate of return.
G = The growth rate of the dividends.
Therefore, the value of the stoc k is Rs.40.04,
calculated as follows: (Rs.3.85)(1.04)
P0 = = Rs.40.04 .14 - .04
Ans, [c].
138. A stock index fund is a mutual fund that mirrors a
stock index. Typically, the fund pays little or no
dividends, and is therefore tax efficient. Option [a] is
incorrec t, nonleveraged equipment leasing
investments have more of an income objective than
a growth objective and are not tax efficient because
the income is taxed each year. Option [b] is incorrect
because option [a] balanced mutual fund usually
invests a large percentage of the fund assets in fixedincome securities. Option [c] is incorrect because
preferred stocks often pay a large dividend and
usually have an income objective.
Ans, [d].
139. Ans, [b].
140. Ans, [a].
141. Ans, [c].
142. Ans, [b].
143. Ans, [b].
144. Ans, [b].
145. Ans, [a].
146. Ans, [a].

Workbook

Investment Planning

73

147. Ans, [b].

184. A retiree at 60 has little appetite for risk as he no


longer has any earning power. Further, he needs
certainty of income. But a small amount of equity is
still recommended as a hedge against inflation. If
not, if he lives until say 80, he will experience a drop
in his standard of living.
Ans, [a].

148. Ans, [b].


149. Ans, [a].
150. Ans, [a].
151. Ans, [c].

155. Ans, [d].

185. It is generally accepted that the order is Bond Fund,


Balanced Fund, Growth Fund and Small Cap Fund.
A small cap fund is more risky than a growth fund.
Earnings of smaller companies are more exposed
to the vagaries of the economic cycle.
Ans, [c].

156. Ans, [d].

186. Ans, [b].

157. Ans, [b].

187. Ans, [b].

158. Ans, [a].

188. Ans, [a].

159. Ans, [b].

189. Ans, [a].

160. Ans, [a].

190. Ans, [b].

161. Ans, [d].

191. Ans, [a].

162. Ans, [c].

192. Ans, [c].

163. Ans, [c].

193. Ans, [c].

164. Ans, [d].

194. Ans, [c].

165. Ans, [d].

169. Ans, [d].

195. A change in the coupon payment from annual to


semiannual will result in the coupons becoming more
significant compared to the final payment on a
weighted average basis and decreases the duration
of a bond . All other factors will increase the duration
of a bond.
Ans, [b].

170. Ans, [d].

196. Ans, [a].

171. Ans, [d].

197. Ans, [b].

172. Ans, [c].

198. Ans, [c].

173. Ans, [a].

199. Ans, [a].

174. By definition, the Treynor Index uses beta as its risk


measure.
Ans, [c].

200. Ans, [b].

152. Ans, [b].


153. Ans, [d].
154. Ans, [c].

166. Ans, [b].


167. Ans, [a].
168. Ans, [d].

175. Ans, [b].

201. Beta=[(S.D of security / S.D of Market) x Coefficient


of correlation]
Ans, [b].

176. Ans, [d].

202. Ans, [d].

177. Ans, [c].

203. Statement III, A strengthening exchange rate will lead


to a drop in competitiveness of a countries exports statement is false. Statement IV is false. Lower
domestic inflation will lead a strengthening exchange
rate. The other statements are true.
Ans, [a].

178. Ans, [a].


179. Ans, [c].
180. Ans, [d].
181. Ans, [b].
182. Ans, [c].
183. Ans, [b].

74

204. Given that the coupon rate is less than the yield to
maturity of similar instruments, the bond will sell at
a discount.
Ans, [b].

Investment Planning

Workbook

205. The calculations are as follows: EPS = 54/200 =


Rs.0.27, PER = 5.00/0.27 = 18.5
Ans, [d].

219. Market Value of a Stock=[(Dividend x (1+growth


rate)] / (Return- growth rate)
Ans, [c].

206. The strong form of market efficiency maintains that


prices contain all information, private and public. The
fact that Mr A was able to beat the BSE is a violation
of the strong form of market efficiency
Ans, [d].

220. Annual rent receivable =300*(1000 x 12) x 0.75 (A)


Annual upkeep & maintenance =Rs 10 lakhs (B)
Net income annually=Rs 17,00,000 (A-B)
Capitalization rate=10%
Value of the complex=Net income / Capitalization rate
Ans, [b].

207. The only answer that does not satisfy II. An increase
in the dividend growth rate leads to an increase in
the stock price
Ans, [c].
208. (7x0.2) + (12x0.3) + (20x0.15) + (5x0.35) = 9.75
Ans, [c].
209. By definition, as the stocks are positively correlated,
they move in the same direction but Stock B will move
three quarter as much as Stock A.
Ans, [c].
210. Beta only captures systematic non-diversifiable risk
of a security. Therefore, I is False, II is True, III is
True, IV is False. Inves tors form portfolios to
eliminate non-systematic risk.
Ans, [b].
211. A money market fund invests only in short term
money market instruments such as short term debt
securities, Treasury bills, banker's certificates of
deposits and bank acceptances. As the instruments
are highly liquid, usually no notice for withdrawal is
required. In fact, some money market funds provide
quasi checking facilities. Money market funds can
pay current income as the instruments mature in the
short term. It should also be mentioned that money
market funds have low interest rate risk because of
the short tenor of the instruments.
Ans, [b].
212. PV is -Rs.20,000, FV is 400,000 with yearly
payments of - Rs.10,000. Number years is 21,
implying retirement age of 51.
Ans, [b].
213. Ans, [a].

221. Ans, [d].


222. Average market capitalization rate: (4.20% + 5.45%
+ 6.00%)/3 = 5.22% Market value: Rs. 15,600/0.0522
= Rs.299,041
Ans, [c].
223. Ans, [d].
224. Using the Growth Dividend Model, the calculations
are as follows:
R = Risk free rate + risk premium
Div1 = Div x (1 +g)
P = Div1/(R - g)
Where P = price
Div1 = dividend in year 1
R = required rate of return or discount rate
g = constant dividend growth rate
R = 0.06 + 0.07 = 0.13
Div1 = 8 x (1 + 0.05) = 8.4
P = 8.4/(0.13 - 0.05) = 105
Ans, [b].
225. As both funds appreciated by the same amount, the
investor made identical returns from both funds:
Rs.4000 x 18% = Rs.720. There is a misconception
that a lower priced fund has a higher return. I is true.
Rs.4000/1.02 = 3921.57 units II is true. Rs.4000/0.58
= 6896.55 units
Ans, [a].
226. Ans, [c].
227. Total return is calculated as follows: (P2 - P1 + Div)/
P1 x 100, (112 - 113 + 6)/113 = 4.4%
Ans, [b].
228. Ans, [c].

214. (100,000) x (1 + 0.015)^(4) - (100,000 x 1.06) =


13.636
Ans, [d].

229. Ans, [a].


230. Ans, [c].

215. Ans, [d].

231. Ans, [c].

216. Ans, [d].

232. Ans, [e].

217. Ans, [c].

233. Ans, [a].

218. Suppose a loan of Rs 1,00,000


Interest = 8%
EMI for 15 Years = Rs 11682.95
EMI for 30 Years = Rs. 8882.74
Rs. 11682.95 < 2 x 8882.74
Ans, [b].

234. Ans, [d].


235. Ans, [a].
236. Ans, [a].
237. Ans, [c].
238. Ans, [c].
239. Ans, [c].

Workbook

Investment Planning

75

240. Ans, [c].

278. Ans, [a].

241. Ans, [b].

279. Ans, [d].

242. Ans, [a].

280. Ans, [b].

243. Ans, [d].

281. Ans, [c].

244. Ans, [c].


245. Ans, [a].

282. Rule of 72, NPER = 72/ rate of compounding, NPER


is no of years to double.
Ans, [e].

246. Ans, [a].

283. Ans, [c].

247. Ans, [b].

284 Ans, [a].

248. Ans, [b].

285. Ans, [b].

249. Ans, [b].

286. Ans, [b].

250. Ans, [a].

287. Ans, [a].

251. Ans, [c].

288. Ans, [c].

252. Ans, [b].

289. Ans, [c].

253. Ans, [a].

290. Ans, [c].

254. Ans, [c].

291. Ans, [c].

255. Ans, [b].

292. Ans, [b].

256. Ans, [c].

293. Ans, [b].

257. Ans, [d].

294. Ans, [a].

258. Ans, [b].

295. Ans, [c].

259. Ans, [b].

296. Ans, [a].

260. Ans, [b].

297. Ans, [b].

261. Ans, [d].

298. Ans, [b].

262. Ans, [b].

299. Ans, [c].

263. Ans, [c].

300. Ans, [d].

264. Ans, [c].

301. Ans, [d].

265. Ans, [a].

302. Ans, [b].

266. Ans, [a].

303. Ans, [c].

267. Ans, [a].

304. Ans, [c].

268. Ans, [a].

305. Ans, [d].

269. (((((4985 / 4000)^(1 / 7)) 1) x100) x 2)


Ans, [a].

306. Ans, [c].

270. Ans, [b].

307. Ans, [b].

271. Ans, [c].

308. Ans, [a].

272. Yield to maturity for bond A:


N = 2, PV = (Rs.900), PMTOA = Rs.60, FV =
Rs.1,000, i = 11.90998
Ans, [d].

309. Ans, [c].

273. Ans, [a].

312. Ans, [c].

274. Ans, [b].

313. Ans, [c].

275. Ans, [a].

314. Ans, [d].

276. Ans, [d].

315. Ans, [a].

277. Ans, [a].

316. Ans, [d].

76

310. Ans, [b].


311. Ans, [a].

Investment Planning

Workbook

317. Ans, [a].

357. Ans, [b].

318. Ans, [a].

358. Ans, [c].

319. Ans, [b].

359. Ans, [c].

320. Ans, [a].

360. Ans, [b].

321. Ans, [c].

361. Ans, [b].

322. Ans, [d].

362. Ans, [b].

323. Ans, [b].

363. Ans, [b].

324. Ans, [b].

364. Ans, [c].

325. Ans, [a].

365. Ans, [c].

326. Ans, [c].

366. Ans, [b].

327. Ans, [d].

367. 10/105 x 100 = 9.52%


Ans, [c].

328. Ans, [a].


329. Ans, [d].
330. Ans, [c].

368. (100-90) + 5 / 90 x 100 = 16.67%


Ans, [b].

331. Ans, [d].

369. PV = 6000, PMT = 100, FV = 7200, NPER = 3


Ans, [c].

332. Ans, [d].

370. Ans, [d].

333. Ans, [a].

371. Ans, [c].

334. Ans, [d].

372. Ans, [b].

335. Ans, [a].

373. 13 x 3 = 39 + 1 x 5 = 44. 44/4 = Rs. 11.00


Ans, [a].

336. Ans, [a].


337. Ans, [a].

374. Yield to maturity =


Bond' s par value - Current bond price

Annual interest +
Number of years to maturity

Bond' s par value + Current bond price

338. Ans, [d].


339. Ans, [b].
340. Ans, [a].

Ans, [b].

341. Ans, [a].


342. Ans, [a].
343. Ans, [a].
344. Ans, [a].
345. Ans, [b].
346. Ans, [b].
347. Ans, [b].
348. Ans, [b].
349. Ans, [c].
350. Ans, [c].

375. The price is calculated as follows: (62 - 52)/52 =


19.2%, 40 x 1.192 = 48
Ans, [a].
376. Expected return = (0.20) (0.05) + (0.40) (0.10) + (.10)
(0.08) + (0.30) (0.11) = 0.091 = 9.1%
Ans, [a].
377.
(a)

Cumulative Wealth Index

CW I5 = 1 (1.07) (1.03) (0.91) (1.06) (1.10) = 1.169


(b)

Arithmetic Mean

351. Ans, [b].

0.07 + 0.03 - 0.09 + 0.06 + 0.10 = 0.034 or 3.4%


5

352. Ans, [b].

= 0.034 or 3.4%

353. Ans, [a].

(c)

354. Ans, [b].

[(1.07) (1.03) (0.91) (1.06) (1.10)]

355. Ans, [b].

= 0.032 or 3.2%

Geometric Mean
1
-1]
5

356. Ans, [a].

Workbook

Investment Planning

77

(d)

Variance

383. The intrinsic value of a bond equals the discounted


value of its promised cash flows. In this case:
Rs. 75
(1.10)1
=
=

Rs. 68.18 + Rs. 61.98 + Rs. 807.66


Rs. 937.82

It is adviced to sell the bond as the price available in


market is more than what is required.
384. i) A bond's YTM is the interest rate that equates the
bond's current price to the discounted value of its
promised cash flows. In this case:

= 54.3
(d)

Standard Deviation
Rs. 1032.40 =

= 7.37
378. This multiple growth DDM expresses a stock's
intrinsic value as :
N

Dt

t
T = 1 (1 + k )

[(k - g) (1 + k)N ]

V = [Rs. 6.00/(1.10)1 + Rs. 6.00/ (1.10)2 + Rs. 7.00/

(1 + y )

Rs.100
Rs.100
+
(1 + y)2
(1 + y )2

If we solve for y, it works out to be 9%. Hence YTM


is 9%.

Rs. 1032.40 =

Rs.100
(1 + y )1

Rs.1200
(1 + y )2

YTC = 12.7%

{Rs. 7.00 x (1.04)}


[(0.10 - .04) x (1.10)4]

= [Rs . 5.45 + Rs . 4.96 + Rs. 5.26] +

{7.28}
[(.06) x (1.4641)]

385. Find the PV of the future cash flows of Rs 800 every


year and final amount at maturity ie 10000. with the
required discount rates.
with 10% YTM=Rs 8771
With a 5% YTM= Rs 12316

= Rs.15.67 + Rs. 82.87 = Rs. 98.54


379. The intrinsic value of a perpetual constant income
stream, discounted at a rate K, is given by:
V = D/K
In the case of H pipes, the intrinsic value of the
preferred stock is:
V = Rs. 12/.15 = Rs. 80
380. If ABC Ltd. will earn 20% on its equity and pay out
50% of its earnings indefinitely, then its growth rate
is
G = r x (1-p)
= .20 x (1-.50)
= .100 = 10.0%
In case of constant growth:
V = [(1-P) x Eo x (1+g)] /(k-g)
Thus for ABC Ltd.
V = [(1-50) x Rs. 4 x (1+.10)] / (.15-.10)
= Rs. 2.20 / 0.5 = Rs. 44
381. F0 = So (1+rf)t 1/12
= Rs. 16 (1.09)
= Rs. 16.115

386. The return on a mutual fund paying a year-end


distribution is given by
rt = [ (NAVt NAVt 1) + Dt] / NAVt-1
In the case of the Saturn fund:
rt = [(Rs.16.90 Rs.18.50) + Rs.1.25] / Rs.18.50 =
0.019 = 1.9%
387. The expected returns on the three securities in
Manoj's portfolio are:
rA = (.30 x -10%) + (.20 x 0%) + (.30 x 10%) + (.20 x
20%) = 4.0%
rB = (.30 - 10%) + (.20 x 10%) + (.30 x 5%) + (.20 x
-10%) = 4.5%
rC = (.30 -0%) + (.20 x 10%) + (.30 x 15%) + (.20 x
5%) = 7.5%
The expected return on Manoj portfolio is therefore:
rp = (.20 -4.0%) + (.50 x 4.5%) + (.30 x 7.5%) =
5.3%

382. F0 = So (1 + rf - d)t = 3500 (1 + .08 - .02)0.5 =


3603.5

78

ii) If the bond can be called in two years for Rs. 1100,
its yield-to-call is found by solving for the YTM assuming
the receipt of only two coupon payments and a call
price of Rs. 1100. That is:

{DN x (1 + g)}

In case of M.A Ltd.

(1.10)3]

Rs.100

The standard deviations of the portfolios three


securities are:
s A = {[.30 x (-10 - 4.0)2] + [.20 x (4-4.0)2] + [.30 x
(10-4.0)2] + [(.20 x (20-4.0)2]}

Investment Planning

Workbook

[58.8 + 3.2 + 10.8 + 51.2] = 11.1%

Jensen Index: Rp - Rf + b P (R m - Rf )
Funda A: 12 [6 + 1.1(5)] = 0.5
Funda B: 10 [6 + 0.9(5)] = 0.5
Funda C: 13 [6 + 1.2(5)] = 1.0
Market Index: 0 (By definition)

s B = {[.30 x (10 - 4.5)2] + [.20 x (10-4.5)2] + [(.30 x


(5-4.5)2] + [(.20 x (-10 -4.5)2]} [9.1 + 6.1 + 0.1 +
42.1] = 7.6%
s C = {[.30 x (0 - 7.5)2 + .20 x (10-7.5)2 + (.30 + (157.5)2 + (.20 x (5 - 7.5)2]} [16.9 + 1.3 + 16.9 + 1.3]
= 6.0%
The fact that the three securities are uncorrelated
with each other simplifies the calculation of the
portfolio's standard deviation. Specially,

391. For A portfolio that receives a contribution at the end


of the measurement period, the rate of return is: R0R
= (End value - Beg value - Cont.) / Beg value
In Vipul's case: RoR = (Rs. 42,000 - Rs. 39,000 Rs. 4000) / Rs. 39000 = - .026 = -2.6%
392.

sA = {(.20)2 x (11.1)2 + (.50)2 x (7.6)2 + (.30)2 x


(6.0)2} [4.9 + 14.4 + 3.2]
= 4.7%
388. The weights that derive the standard deviation of
portfolio to zero, when the returns are perfectly
negatively correlated, are:
Wp =

30
sQ
=
= 0.545
sP + sQ
25 + 30

The expected return of the portfolio is:


0.545 16% + 0.455 18% = 16.91%
389. (a) Covariance (A, B) = AB * s A * s B
= 0.60 x 15 x 8 = 72
(b) Expected return = 0.6 x 16 + 0.4 x 12 = 14.4%
Risk (standard deviation) =
[X2A s2A + X2B s2B + 2 *wa*wbCov (A, B) ]
= [(0.6)2 x 225 + (0.4)2 X 64 + 2 X.6*.4 72]

The mean risk free rate was 10%. Calculate Treynor


measure, Sharpe measure and Jensen measure.
Answer
Treynor
Sharpe Ratio
P = 25%
P = 5.55
Q = 29.16%
Q = 6.36
R = 33.3%
R = 7.5
Market = 30%
Market = 6
Jensen
P = -0.4
Q = 0.4
R = 1.8
Market = 0
393. Ans, [b].

= 11.21%
390. Treynor measure:

Rp - Rf
bp

12 - 6
Fund A:
= 5.45
1.1

394. Ans, [d].


395. Ans, [d].
396. Ans, [c].

10 - 6
= 4.44
0.9

397. Ans, [c].

13 - 6
Fund C:
= 5.83
1.2

399. Ans, [a].

Fund B:

11 - 6
1 .0

market Index:
Sharpe Measure:

Rp - Rf
sP

12 - 6
Fund A:
= 0.333
18
Fund B:

10 - 6
= 0.267
15

13 - 6
Fund C:
= 0.350
20
Market Index:

Workbook

11 - 6
= 0.294
17

398. Ans, [b].

400. Ans, [a].


401. Ans, [b].
402. Ans, [e].
403. Ans, [e].
404. Ans, [c].
405. Ans, [a].
406. Ans, [a].
407. Ans, [b].
408. Ans, [c].
409. Ans, [e].

Investment Planning

79

410. Ans, [b].

450. i Ans, [a]

ii Ans, [b]

iii Ans, [a].

411. Ans, [a].

451. i Ans, [c]

ii Ans, [a]

iii Ans, [b]

412. Ans, [b].

iv Ans, [c].

413. Ans, [d].

452. i Ans, [d].

ii Ans, [a].

453. i Ans, [c].

ii Ans, [d]

iii Ans, [c].

454. i Ans, [e].

ii Ans, [b]

iii Ans, [b].

455. i Ans, [e].

ii Ans, [d]

iii Ans, [d].

456. i Ans, [d].

ii Ans, [c]

iii Ans, [d].

457. i Ans, [b].

ii Ans, [b]

iii Ans, [c].

458. i Ans, [d].

ii Ans, [e]

iii Ans, [d].

459. i Ans, [d].

ii Ans, [c]

iii Ans, [b].

422. Ans, [c].

460. i Ans, [c].

ii Ans, [d]

423. Ans, [d].

461. i Ans, [b].

ii Ans, [a]

iii Ans, [d].

462. i Ans, [a].

ii Ans, [b]

iii Ans, [c].

463. i Ans, [b].

ii Ans, [a]

iii Ans, [a].

ii Ans, [d]

464. i Ans, [d].

ii Ans, [b]

iii Ans, [d].

426. i Ans, [b]

ii Ans, [d].

465. i Ans, [c].

ii Ans, [b]

iii Ans, [c].

427. i Ans, [c]

ii Ans, [c].

466.

428. i Ans, [c]

ii Ans, [d].

429. i Ans, [c]

ii Ans, [a].

430. i Ans, [c]

ii Ans, [a].

431. i Ans, [c]

ii Ans, [c].

432. i Ans, [d]

ii Ans, [b].

iii Ans, [b].

433. i Ans, [c]

ii Ans, [b].

iii Ans, [d].

434. i Ans, [b]

ii Ans, [b].

435. i Ans, [a]

ii Ans, [b].

436. i Ans, [a]

ii Ans, [b]

437. i Ans, [c]

ii Ans, [d].

438. i Ans, [b]

ii Ans, [b].

439. i Ans, [c]

ii Ans, [c].

iii Ans, [b].

467. i Ans, [c]

ii Ans, [c]

440. i Ans, [c]

ii Ans, [b]

iii Ans, [c].

468. i Ans, [d]

ii Ans, [b].

441. i Ans, [a]

ii Ans, [b]

iii Ans, [b].

469. i Ans, [d]

ii Ans, [b].

442. i Ans, [b]

ii Ans, [d]

iii Ans, [b].

470.

443. i Ans, [b]

ii Ans, [c]

iii Ans, [a].

444. i Ans, [d]

ii Ans, [c]

iii Ans, [d].

445. i Ans, [d]

ii Ans, [c]

446. i Ans, [a]

ii Ans, [b]

414. Ans, [d].


415. Ans, [d].
416. Ans, [b].
417. Ans, [d].
418. Ans, [d].
419. Ans, [a].

iv Ans, [d].

420. Ans, [c].


421. i Ans, [c]

ii. Ans, [d]

iii. Ans, [c].

Iv. Ans, [d].

424. i Ans, [c]

ii Ans, [b]

425. i Ans, [d]

ii Ans, [b]

iv Ans, [b]

iv Ans, [b]

(i)
iii Ans, [b].

Rs.38 for purchasing stocks but out of this Rs. 3


is received from writing call, so the net cash flow
is cash outflow of Rs. 35 for call writer.

Ans, [b].
(ii)

Rs.2 from stock price and Rs. 3 from writing


the call.

Ans, [c].
(iii) 1 / 38 = 2.63%
Ans, [d].

iii Ans, [c].

(iv) 2 / 38 = 5.26%
Ans, [a].
iii Ans, [b].

iii Ans, [d]

iii Ans, [a].

v Ans, [a]

447. i Ans, [c]

ii Ans, [b].

448. i Ans, [d]

ii Ans, [c].

449. i Ans, [c]

ii Ans, [d].

80

iii Ans, [d].

Investment Planning

Workbook

i. The return on Ray's stock portfolio during the year:


Appreciation of the stock - Rs. 25,000 - Rs. 20,000 =
Rs. 5,000
Dividend receipt = Rs. 1,000
Total return after one year = Rs. 6,000 i.e. Rs. 6,000
for Rs. 20,000 which works out to 30%.
ii. The return on Ray's bond portfolio during the year
Depreciation of the bond = Rs. 30,000 - Rs. 23,000 =
Rs. 7,000 Coupon receipt = Rs. 3,000 Total return after
one year = Rs. 4,000 i.e. Rs. - 4,000 for Rs. 30,000
which works out to - 13.33%.
iii. The return on Ray's total portfolio during the year.
Depreciation of the portfolio - Rs. 50,000 - Rs. 48,000
= Rs. 2,000 Dividend/Coupon receipts = Rs. 4,000
Total return after one year = Rs. 2,000 i.e. Rs. 2,000
for Rs. 50,000 which works out to 4%.
471. i. The weight of Security B: The mean between the
cost price and expected year end price = (78 + 75) /
2 = 76.5 The difference between the mean and the
cost price (initial price) = Rs. 1.5. Ratio between the
difference and the cost price = weight of security =
2%
ii. The expected return on Security C. The ratio
between appreciation and the cost price determines
the rate of return. i.e. Rs. 15 per share on the cost
price of Rs. 125 per share - works out to 12%
iii. The return on Security A Similar to (ii) above, the
ratio between cost price and expected price - i.e.
40/100 = 40%
iv. The figure of return on the portfolio that is approx.
equals in the following: Similar to (ii) & (iii) above,
the return of

Price earning ratios when considered along side


historical evidence for the same stock, sector or
market, if the ratio is above its historical average,
pric e may be c ons idered too high unless a
substantial increase in earnings is likely in the
following years. So, investment in Acharya is a better
choice for investment with the lower ratio.
473. i. ESTIMATED Dividend = Rs. 1.5 Growth Rate =
11% (estimated) Discount Rate (assumed) = 15%
Present Value of the share = P0 = Expected Div./
(discount rate - growth rate) Expected Dividend =
Current dividend x growth rate - i.e. 1.5 x 111%
(100+11) = 1.665 Present Value = 1.665/15 - 11 =
1.665/4% = 1.665/0.04 = 41.625.
ii. Real Term (reality) Dividend = Rs. 1.5 Growth
Rate = 7% (reality) Discount Rate (assumed) = 15%
Expected Dividend = Current dividend x growth rate
- i.e. 1.5 x 107% (100+7) = 1.605 Present Value =
1.605 / 15-7 = 1.605/8% = 1.605/0.08 = 20.063
In reality, it may be seen that the present value of
the share is only Rs. 20 whereas the current share
price is Rs. 25. Only on estimated growth rate of
11% (as against a real growth rate of 7%) the share
price may touch Rs. 42. On the estimation of real
terms, it is better to sell.
Ans, [b].
(Note : If the es timation is bas ed on s ound
extraneous factors adding to the growth, that the
condition c an be deemed to be acc eptable for
holding or buying)
474. Ans, [b]
475. Jensen's Index: Formula: - Fund's mean - [Risk free
rate of Market Index + (difference a the market index
mean - free rate) x Fund's beta] FUND KASHYAP a
14% - [9% + (12% - 9%) x 1] = 14% - 12% = 2%
FUND VIMALESH a 16% - [9% + (12% - 9%) x 2] =
16% - 15% = 1%
Treynor index: Formula: - (Fund's mean - free of
Market index) / fund's beta FUND KASHYA a (14% 9%) / 1 = 5 FUND VIMALESH a (16% - 9%)/2 = 3.5

472. i. The dividend yield of Acharya Dividend yield =


(Dividend per share x 100) / Market price = (0.12 x
100) / 40 = 0.3
ii. The P/E Ratio of Bhushan P.E. Ratio (Price
Earning Ratio) = Market price/Earnings per share
Earning per share = (Profit after tax - Div. on Pref.
Shares)/no. of equity shares Earnings per share is
given. So, P/E Ratio = 60 / 0.175 = 342.86 i.e. 343
iii. The better investment proposition of the two: P/E
Ratio of Bhushan = 343 (shown above) P/E Ratio of
Acharya = 40/0.22 = 181.88 i.e. 182 Please note "Lower the denominator, higher the ratio."

Workbook

Sharpe Index: Formula: - (Fund's mean - free rate


of Market index)/Std. Deviation of the Fund
FUND KASHYAP a (14% - 9%)/12 = 41.67%
FUND VIMALESH a (16% - 9%) / 18 = 38.89%
MARKET a (12% - 9%)/9 = 33.35%
476. Use pv function on excel or This should be dealt
with the discounted cash flow method. Present value
= PMT/(1+i)1 + PMT/ (1+i)2 + PMT / (1+i)3 + PMT
(1+i)4 + PMT/(1+i)5 + PMT / (1+i)6

55,000
(1 + 0.15)1
55,000
(1 + 0.15)4

Investment Planning

55,000
(1 + 0.15)2

55,000
(1 + 0.15)5

55,000
(1 + 0.15)3

55,000
(1 + 0.15)6

8,50,000
(1 + 0.15)6

81

= 47,826 + 41,588 + 36,163.4 + 31,446.4 + 27,344.7


+ 23,778 + 3,67,478.5 = 5,75,625
Ans, [c].
477. Ans, [a].
478. The risk on the portfolio takes into consideration the
standard deviations of individual securities and the
interactive risk between the pair of securities.
FORMULA:XA = 60% XB = 40% s A =14.5%
s B = 18.5% r AB = 0.91 (correlation coefficient)

s xy = [X2A s 2A + X2B s 2B + 2XA XB s A s B r AB]


= square root of [0.62 x 14.52 + 0.42 x 18.52 + 2
x 0.6 x 0.4 x 14.5 x 18.5 x 0.91]
= sq. root of 75.69 + 54.76 + 17.17
= sq root of 247.62 = 15.74
479. i. Calculate expected rate of return on the portfolio.
Expected return on security A=(0.1 x 40)+(0.2 x
20)+(0.4 x 0) + (0.2 x -5) + (0.1 x -10) = 6%
Expected return on Security B = (0.1 x 40)+(0.2 x
30) + (0.4 x 15)+(0.2 x 0) + (0.1 x -20) = 14%
Then, calculate the standard deviation for both
securities.
Security A = v (0.1 x (40-6)2 + (0.2 x (20-6)2 + (0.4 x
(0.6)2) + (0.4 x (0-6)2) + (0.2 x (-5-6)2) + (0.1 x (-106)2)) = v ((0.1 x 1156) + (0.2 x 196) + (0.4 x 36) +
(0.2 x 121) + (0.1 x 256)) = v 219 = 14.8%
Security B = v ((0.1 x (40-14)2) + (0.2 x (30-14)2) +
(0.4 x (15-14)2) + (0.2 x (0-14)2) + (0.1 x (-20-14)2))
= v (0.1 x 676) + (0.2 x 256) + (0.4 x 1) + (0.2 x 196)
+ (0.1 x1156) = v 274 = 16.55%
ii. Further, calculate the covariance between the
two securities. Assumption: Equal amount invested
in both securities since so proportion is assigned.
Covariance : = (0.1 x (40-6)(40-14)+(0.2 x (20-6)(3014) + (0.4 x (0-6)(15-14) + (0.2 x (-5-6) (0-14) + (0.1
x (-10-6)(-20-14) = 216
iii. Further calculate the standard deviation for the
portfolio risk: Std. Deviation:
= [(0.5 x 14.82 + 0.5 x 16.552) + (2 x 0.5 x 0.5 x
216)] = [109.52 + 136.95 + 108] = [354.47] =
18.83
480. Ans, [a].
481. Ans, [b].
482. Ans, [a].

489. Use the PRICE Function in excel. Eg. For Sett. Date
use 4/1/2004 (mm -dd-yyyy). Maturity Date: 3/31/2004;
Rate = 20; Yield = 8%; Redemption Value = Rs. 100;
Frequency = 1; Hence Price is Rs. 110.41.
Ans, [b].
490. Duration of a level annuity is: (1 + Yield/Yield) CE
No. of payments 91 + yield) ^ no of payments - 1
Hence duration = 2.88 years
Ans, [a].
491. Use the formula - P0 = D1/(R-G) ; Where P0 = 40;
D1 = 4.00; R = 12%; Hence G = 2.00%
Ans, [a].
492. Beta of Infocomm is (25-3)/(15-5) = 2.2; Beta of
FMCG = (12-7)/(15-5) = 0.5
Ans, [d].
493. Treynor Ratio = (Rp - Rf) / Beta; Hence 1.33; Sharpe
Ratio = (Rp CE Rf)/Standard Deviation; Hence 0.04;
Jensen = Rp - [Rf + B (RM CE RF)]; It measures the
risk adjusted portfolio return. It is also known as
JensenTMs Alpha; Hence - 4.25
Ans, [b].
494. 15.3%
495. 14.4%
496. i) 13.6% (ii) 1.5 Beta = (20%-8%)/ 8%
497. 16%
498.

i. Assuming zero risk free returns the sharpe ratio


for all the above is 2,1.6,1,1,1.8,1.33. Stock A is of
high returns for a unit of risk if the expected return is
8%, If the expected return is higher then stock E is a
good choice as it gives 9% with slight higher risk. If
the expected returns is higher and is at 12%, The
only choice is to go for higher risk and select stock C.
ii. For a portfolio of 75% A and 25%B The expected
returns is (.75 x 8%+.25 x 12%)=9% while the
Standard deviation for portfolio is 6%. It is not a good
choice as stock E gives same returns at a standard
deviation of 5%. It may be preferable to go for choice
of stock E than the above portfolio.
499. i)

16.2% & 16.2%

483. Ans, [d].

ii) -8.62%, 21.60%, 39.76%, 3.51%, 24.73%

484. Ans, [c].

iii) 16.2%

485. Ans, [b].


486. The length of time the investor expects to hold the
investment
Ans, [a].
487. Ans, [a].
488. PPF investments can only be made on behalf of
immediate family members.
Ans, [b].

82

500. i) VV has lower return and caries higher risk, hence


investing in XY is better than investing in VV.
ii) 22.6%, 37.06%
iii) 22.76%, 17.5%
iv) 0.974

Investment Planning

Workbook

Notes

Workbook

Investment Planning

83

Notes

84

Investment Planning

Workbook

You might also like